Math 251 diary, fall 2010
Later material
Much later material
In reverse order: the most recent material is first.


Thursday, October 7, sections 12-14, and Friday, October 8, sections 15-17, lecture #11

Review sessions for the first exam

All students in all sessions are welcome at either or both review sessions. I thank both of these gentlemen sincerely.

A possible "test item" for Calc 1
Suppose that W is a function of 1 variable and we know that W(0)=A and W´(0)=B and W´´(0)=C. If Q is another function of one variable defined by Q(v)=5(W(v))2–8W(v)+7, then compute
    Q(0), Q´(0); Q´´(0).

This is certainly a legal question for a calc 1 course after the Chain Rule and Product Rule has been covered. Let's solve it.

The same sort of thing can be done in more than 1 variable, and things get rather confusing fast.

Redoing (and extending!) the QotD from last time
Suppose I have a function of two variables, f(x,y), and I know the following information:
    f(0,0)=A and (∂f/∂x)(0,0)=B and (∂f/∂y)(0,0)=C and
    (∂2f/∂x2)(0,0)=D and (∂2f/∂x∂y)(0,0)=E and (∂2f/∂y2)(0,0)=F
Now I will define a function g with a one-variable input by the equation g(t)=f(4t1+5t2,6t3). 1 2 3 4 5 6 This is entirely absurd -- I just assigned the powers and the coefficients in a direct way.

Some questions and some answers

This ends the portion of the lecture devoted to material that may be tested on our first exam. The remainder of the lecture is in the next section of the diary.


Tuesday, October 5, sections 12-14 and sections 15-17, lecture #10

Review sessions for the first exam

All students in all sessions are welcome at either or both review sessions. I thank both of these gentlemen sincerely.

I asked students to do several problems at the board.

A normal line
Suppose we have a surface in space defined by x3z+y2z+xyz=7 Notice that the point (2,–1,1) is on this surface. Find parametric equations for a line normal or perpendicular to the surface at this point.

Here's a solution: let f(x,y,z)=x3z+y2z+xyz. The ∇f is always perpendicular to level surfaces. We can compute ∇f: it is <3x2z+yz,2yz+xz,x3+y2+xy>. At (2,–1,1), ∇f is <13,0,7>. So we need parametric equations and we know that the point (2,–1,1) is on the line and the vector <13,0,7> is in the direction of the line. Therefore we can write parametric equations:
   x=13t+2
   y=0t–1
   z=7t+1

This problem was enthusiastically (?) solved by Mr. Selwyn Joy and Mr. Siva Yedithi and Mr. Daren Tang and Mr. Ikshit Gandhi, and I thank them.

Another example
Suppose I want an equation for the plane tangent to z=3x2+5xy2 when x=1 and y=2. Well (curious trick!) if I consider T(x,y,z)=3x2+5xy2–z then the surface z=3x2+5xy2 is exactly the isothermal or level surface corresponding to T=0. Now ∇T will be perpendicular to that surface. We compute: ∇T=<6x+5y2,10xy,–1>. We know that x=1 and y=2 so that the gradient is <26,20,–1>, which is a vector perpendicular to the plane. Now we need a point on the surface. But we know z=3x2+5xy2 and x=1 and y=2 so we can deduce z: z=23. A point on the plane is (1,2,23). Therefore an equation for the tangent plane is
    26(x–1)+20(y–2)+–1(z–23)=0.

This problem was enthusiastically (??) solved by Ms. Barbara Dudycz and Mr. Elie Rosen and Mr. Jeremy Priestner and Mr. Joseph Dziuba. Again, these students should be thanked.

One further nice use of gradient is this.

Directional derivative
If u is a unit vector, then the directional derivative of T at (x,y,z) in the direction u is the rate of change of T at unit speed in the direction u (at the point). The textbook's notation for this is DuT(x,y,z) and the discussion earlier should convince you that the directional derivative's value is ∇T(x,y,z)·u.

Please notice that ||∇T(x,y,z)·u|| is ||∇T(x,y,z)|| multiplied by ||u|| (that's always 1 since u is a unit vector) multiplied by cos(θ), where θ is the angle between u and ∇T(x,y,z). But –1≤cos(θ)≤1. Where is it +1 and where is it –1? A little thought tells me this:

So for example, if f(x,y,z)=x3z+y2z+xyz and if we were interested in how this function changed at the point (2,–1,1) then a unit vector in the direction of greatest increase is <13/sqrt{208},0,7/sqrt{208}> and the amount of the increase is sqrt{208}
I hope that 208 is 132+02+72!

This is all amazing to me, since the computations are rather straightforward and the information obtained is rather specific but without the "technology" of the gradient, I don't see any simple strategy to get the directions of greatest increase and decrease. This idea is used computationally a great deal in the "real world".

A sphere problem
The points in R3 which satisfy x2+y2+z2=R2 are a sphere of radius R centered at (0,0,0). We could say that z is defined as a function of x and y by this equation. In fact, since the situation is relatively simple, we can actually solve for z in terms of x and y: z=±sqrt(R2–x2–y2). From this equation we can compute ∂z/∂x etc. But if we assume that z is defined implicitly as a function of x and y by the equation, we can compute using the Chain Rule by just ∂/∂x'ing the whole equation:
    x2+y2+z2=R2 becomes
    2x+0+2z(∂z/∂x)=0 so ∂z/∂x=–2x/2z.

Some of the details of this computation need explanation. The derivative with respect to x of x2 is certainly 2x. What about the next term, the 0? x and y are independent variables, and there is no way a change in x can force changes in y. So here ∂y/∂x is 0. (The notation in this subject is notoriously unhelpful -- you must keep track of the logical meaning of the symbols.) And the Chain Rule applies to z2: we are assuming that z is a differentiable function of x and y, so we apply the Chain Rule to the square of this unknown function, and the result is twice the function multiplied by the function's derivative. Last, on the right-hand side of the equation, R2, in spite of its appearance, is a constant and therefore has derivative equal to 0. We take the resulting equation and "solve" for the desired ∂z/∂x.

 
A more complicated example
The computations can be intricate. Let me try to write something quite irritating:
Suppose z is implicitly defined as a function of x and y by the
equation z2sin(x+y2)+z3x=5ezy. Find a formula for ∂z/∂y.
What I'll do is ∂/∂y the whole equation carefully. Here x and y are independent, so ∂x/∂y is 0. The differentiation is a wonderful (?) example of expression swell. I'll use zy instead of ∂z/∂y so I can write less:
    2z(zy)sin(x+y2)+z2cos(x+y2)(2y)+3z2(zy)=5ezy(zyy+z)
Let me try to analyze this intricate computation: for the z2sin(x+y2) term I use the Product Rule followed by the Chain Rule on each of the factors. I use the Chain Rule on z3x and realize that x is a "constant" for this computation. On the other side, 5ezy needs first the Chain Rule, and then the Product Rule on its "argument", zy. The game now is to "isolate" and solve for zy in this mess. Let me rewrite the equation:
    2z(zy)sin(x+y2)+z2cos(x+y2)(2y)+3z2(zy)=5ezy(zyy+z)
Maybe that typographical change helps. So I rewrite:
    zy(2z sin(x+y2)+3z2)+z2cos(x+y2)(2y)=zy(5ezyy)+5ezyz.
Now I get, after some rearranging and a division:
    (z2cos(x+y2)(2y)+5ezyz)
zy= ------------------------
    (2z sin(x+y2)+3z25ezyy)
What a mess! This computation has no extra redeeming social value -- it is what it is.
 

Crazy computations in thermodynamics and physical chemistry
I wanted to briefly indicate some horrible computations which come up in applications. The computations are not really horrible, but the notation makes them look quite weird. First, a bit of preparation. If we have F(x,y) and both x and y are functions of one variable v (so x=x(v) and y=y(v)), then what happens if we try to differentiate F with respect to v? So basically we have, say, g(v)=F(x(v),y(v)), and I want to understand g´(v). The control from v to F's output is passed through two variables. I remember that F(x+Δx,y+Δy)≈F(x,y)+(∂F/∂x)Δx+(∂F/∂y)Δy so that F(x+Δx,y+Δy)–F(x,y)≈(∂F/∂x)Δx+(∂F/∂y)Δy. If I now divide by Δv, we get ΔF/Δv≈(∂F/∂x)(Δx/Δv)+(∂F/∂y)(Δy/Δv). If I take limits as Δv→0, then this formula appears:
    g´(v)=(∂F/∂x)x´(v)+(∂F/∂y)y´(v).
This is another "Chain Rule" and the weird thing about it is the + which comes from the definition of differentiability in several variables. So let me show you some applications.

Implicit functions, two dimensions
I first began with a return to a 1 variable calculus situation:
Suppose F(x,y) is a differentiable function of 2 variables, and the equation F(x,y)=0 defines y implicitly as a function of x. What is dy/dx in terms of F and "things" related to F?
So take the equation F(x,y)=0 and d/dx this equation. The right-hand side is 0, and the left gives you:
     ∂F/∂x(dx/dx)+∂F/∂y(dy/dx) by the chain rule.
Certainly dx/dx is 1, and dy/dx is what we want, so we can "solve" for it in the equation ∂F/∂x+∂F/∂y(dy/dx)=0. This means:

A formula!

dy      ∂F/∂x 
-- = – ------- 
dx      ∂F/∂y

Example
I think an example is needed here before we go on. Let's look at a Calc 1 problem:
    Find dy/dx if y3–7xy2+4x5–6=0.

Calc 1 solution to Calc 1 problem
We d/dx everything, being careful to remember that y=y(x) mysteriously. Then:
3y2y´(x)–7y2–(7x)2yy´(x)+20x4=0, and now we solve for y´(x). We get:
y´(x)(3y2–(7x)2)–7y2+20x4=0 so that y´(x)=–(–7y2+20x4)/(3y2–(7x)2).

New technology (?) solution to Calc 1 problem
We will use the formula above. Here F(x,y)=y3–7xy2+4x5–6 so that
∂F/∂x=–7y2+20x4 and ∂F/∂y=3y2–(7x)2y+0 and the formula gives
dy/dx=–(∂F/∂x)/(∂F/∂y)=–(–7y2+20x4)/(3y2–(7x)2y+0) which is of course the same answer! And you can look at see the same pieces occurring, so the world is not so crazy.

The darn formula, though, is a bit mysterious. If you try to understand the form (?) of the formula, the ∂x and ∂y might seem in the wrong place and there might be an extra minus sign ... and ... and ... the notation is terrible!

P and V and T
Do you know about gas laws? For a gas, there are the quantities P (pressure) and V (volume) and T (temperature). A gas law might be a function of three variables which relates these quantities:
     G(V,P,T)=0.
If we assume that the function is differentiable and that each one of the quantities is implicitly defined as a function of the other two by the function, something funny happens. Let me show you.

Suppose that G(V,P,T)=0 implicitly defines V as a function of P and T. Let's compute ∂V/∂P. Here T is constant, and sometimes in thermodynamics the quantity is called (∂V/∂P)T just to remind people that T is constant. We will ∂/∂V the equation G(V,P,T)=0.
I use the chain rule, and the result is:
(∂G/∂V)(∂V/∂P)+(∂G/∂P)(∂P/∂P)+(∂G/∂T)(∂T/∂P)=0.
But ∂P/∂P must be 1 (the derivative of something with respect to itself) and ∂T/∂P must be 0 (because T is constant!). Therefore we can solve for ∂P/∂V just as we got dy/dx before and get:
    ∂V/∂P=–(∂G/∂P)/(∂G/∂V).
By the way, in applications people frequently change ∂V/∂P to (∂V/∂P)T to help remember that T is constant in this computation.

So far so good. But in fact we can find other partials in a similar way:
    ∂P/∂T=–(∂G/∂T)/(∂G/∂P)
    ∂T/∂V=–(∂G/∂V)/(∂G/∂V).
Now clearly (NOT AT ALL CLEARLY!):
    (∂V/∂P)T(∂P/∂T)V(∂T/∂V)P=–1
because when we multiply all these expressions together the fractions all cancel and we are left with –1. Why is this true physically and what does it mean? Take physical chemistry, take thermo, etc., and find out. But the notation is horrible and, for me, makes things harder to state and understand.

 
Some students expressed doubts about all this. So here is an explicit example. My "gas law" will be silly and not physically meaningful.

Suppose P and V and T obey the following "law":
    P2T+Ve3P+5T=0.

So the left-hand side of that equation is G(P,V,T). Now what is ∂V/∂P (with T held constant). So ∂/∂P the equation. The result is 2PT+(∂V/∂P)e3P+5T+Ve3P+5T3=0. We solve to get ∂V/∂P=(2PT+Ve3P+5T3)/(e3P+5T).

Now I want ∂P/∂T (with V held constant). So ∂/∂T the equation. The result is 2P(∂P/∂T)T+P2+Ve3P+5T{3(∂P/∂T)+5}=0. We solve to get here ∂P/∂T=(P2+Ve3P+5T5)/(2PT+Ve3P+5T3).

Finally, I will try to compute ∂T/∂V with P held constant. We ∂/∂V the equation, and we get P2(∂T/∂V)+1e3P+5T+ Ve3P+5T5(∂T/∂V)=0. Now solving gives ∂T/∂V=(1e3P+5T)/(P2+Ve3P+5T5).

Now plug all this stuff into (∂V/∂P)T(∂P/∂T)V(∂T/∂V)P. Here we go:
((2PT+Ve3P+5T3)/(e3P+5T) )((P2+Ve3P+5T5)/(2PT+Ve3P+5T3) )((1e3P+5T)/(P2+Ve3P+5T5) ).
Check it out! Lots of things cancel, and we have (–1)3=–1. YES!!!

More abstractly ...
I could analyze a function of two variables defined by using a function of one variable (!?). O.k., here is what I mean: suppose I have a function f with one variable input and one variable output. So maybe we could write f as f(w), where w is a number. I could define F(x,y)=f(x2+y2) and then try to compute, say, both ∂F/∂x and ∂F/∂y. Since I don't know much about f, when I need to differentiate it I just will write f´ and try to go on. So here we go:
    ∂F/∂x=f´(x2+y2)2x.
    ∂F/∂y=f´(x2+y2)2y.
But then y∂F/∂x–x∂F/∂y=0 because the f´'s cancel. So if I wanted to solve the partial differential equation y∂F/∂x–x∂F/∂y=0 I have a collection of solutions:
    sin(x2+y2) and ex2+y2 and 56(x2+y2)332 are all solutions.
Since such equations arise often in practice, this is nice.
 

A more significant example
If we take a long "homogeneous" rope, and wiggle it a bit, the wiggles propagate down the rope. It turns out (neglecting units, neglecting certain other hypotheses, but keeping the central idea) that for small wiggles, if we define f(x,t) to be the height of the rope at position x and at time t, then
    ∂2f/∂x2=∂2f/∂t2.
This is called the one-dimensional wave equation. I can tell you about all of the solutions. Suppose L and R are both functions of one variable. Then define f(x,t)=L(x+t)+R(x–t). I'm not going to tell you anything about the functions L and R except that they are differentiable. Then the Chain Rule again applies:
    ∂f/∂x=L´(x+t)(1)+R´(x–t)(–1) and further ∂2f/∂x2=L´´(x+t)(12)+R´´(x–t)(1)2.
No, I don't really need to "write" the 1's but I want you know that they are there: the Chain Rule was used. Now again:
    ∂f/∂t=L´(x+t)(1)+R´(x–t)(–1) and further ∂2f/∂t2=L´´(x+t)(1)2+R´´(x–t)(–1)2.
And clearly (no!) f(x,t) satisfies the Wave Equation. The names of the two functions, L and R, were chosen because they model the movement of two "waves", a wiggle to the Left and a wiggle to the Right. I couldn't show this in class because I left the darn bungee cords in the car: sorry. So you beed to imagine ths situation or try a demonstration yourself (take a long cord, put a friend at the other end, and both of you should wiggle the cord).

To the right is a picture created by Maple of the function f(x,t)={1/(1+.25(x–t)2)}+{.3/(1+.4(2+x+t)2}. The picture shows this function over the interval [–12,12] on the x-axis, and t varies from –7 to 7. The picture was created using the animate command.
The wave traveling to the right has profile given by R(v)=1/(1+.35v2) and the wave traveling to the left has profile given by L(v)=.3/(1+.4(2+v)2). I hope you enjoy watching them.

 
Why?
All this is not intuitively obvious, at least to me. I bet that some engineers and physicists would assert that the equation is intuitive and clear. The equation turns out to be a very good model of physical vibrations, for at least small displacements (just like Hooke's Law does describe springs, but not if you try to stretch an ordinary rubber band 10 feet!). Here are two links to reasoning which shows how this equation follows from physical ideas:
     At the University of British Columbia This uses Newton's Law and force considerations.
     A Wikipedian entry This uses Newton's Law and Hooke's Law more directly.

The two-dimensional wave equation is ∂2f(x,y,t)/∂x2+∂2f(x,y,t)/∂y2=∂2f(x,y,t)/∂t2. It describes vibrations in thin plates.

The three-dimensional wave equation is ∂2f(x,y,z,t)/∂x2+∂2f(x,y,z,t)/∂y2+∂2f(x,y,z,t)/∂z2=∂2f(x,y,z,t)/∂t2 describes vibrations in solid objects.

It is much more difficult to find simple interesting solutions for the two and three dimensional equations than for the one-dimensional equation.
 

A large part of the exam will ask about the material we discussed in both of the last two lectures. This material is important and it summarizes much of what we've done.

The QotD
Suppose I have a function of two variables, f(x,y), and I know the following information:
    f(0,0)=A and (∂f/∂x)(0,0)=B and (∂f/∂y)(0,0)=C and
Now I define a function g with a one-variable input by the equation g(t)=f(4t1+5t2,6t3). 1 2 3 4 5 6 This is entirely absurd -- I just assigned the powers and the coefficients in a direct way.

What is g´(0)?
Since g(t)=f(4t+5t2,6t3), I will use the Chain Rule and the fact that f is differentiable in two variables. That is,
    g´(t)=(∂f/∂x)(4t+5t2,6t3)(4+10t)+(∂f/∂y)(4t+5t2,6t3)(18t2).
If we plug in t=0, then we get g´(0)=(∂f/∂x)(0,0)(4)+(∂f/∂y)(0,0)(0)=4B.


Thursday, September 30, sections 12-14, and Friday, October 1, sections 15-17, lecture #10

A key definition
f(x,y) is differentiable at (x,y) if there are numbers Constant1 and Constant2 so that for h and k small, f(x+h,y+k)=f(x,y)+Constant1h+Constant2h+Error, where the Error term→0 faster than |h|+|k| (so, faster than first order -- it is H.O.T.).

Important results
Before hysteria strikes, here are two results which are verified in the text. They are not difficult to check (again, the essential key is the 1 variable Mean Value Theorem), but we just don't have time in class.

Theorem If f(x,y) is differentiable, then the partial derivatives of f(x,y) exist, and Constant1=∂f/∂x(x,y) and Constant2=∂f/∂y(x,y).

Theorem If ∂f/∂x and ∂f/∂y are both continuous then f(x,y) is differentiable (in the approximation sense defined above).

Please realize that essentially all functions we will meet in Math 251 will satisfy the hypotheses of the preceding theorem, so these functions will be differentiable and will have suitable approximation properties.

In one variable, we saw that differentiable was the same as writing f(x+w)=f(x)+Qw+Error where Error→0 faster than first order as w→0. The idea of linear approximation in one variable takes advantage of the fact that the Error will be small, smaller eventually than any constant multiple of |w| (that's a very strong qualitative statement!). So linear approximation in 1 variable is f(x+w)≈f(x)+Qw. Geometrically, as you may remember from calc 1, the idea is that the true value of f at x+w is replaced by the tangent line's value at x+w. A picture of this situation, which I hope is familiar to you, is shown to the right.
To the right is an attempt at a picture of the two-dimensional situation. A piece of the graph z=f(x,y), a surface, is shown. Also shown is the plane which is tangent to this surface at (x,y,f(x,y)). In this case the tangent plane lies above the surface. Then the domain point (x,y) is perturbed to (x+h,y+k), and the picture attempts to show the true value (on the surface) above this point, and the height above (x+h,y+k) on the tangent plane. Now differentiability for a two-variable function is f(x+h,y+k)=f(x,y)+(∂f/∂x)h+(∂f/∂x)h+Error, It turns out that if we drop the Error which is the "stuff" that →0 faster than first order, then the approximation f(x+h,y+k)≈f(x,y)+(∂f/∂x)h+(∂f/∂x)h is called the linear approximation and it is exactly the distance "up" to the tangent plane.

Linear approximation: a (perhaps silly) example
Here we looked at something like f(x,y)=sqrt(x4–y2+2xy–3). Notice that f(2,3)=sqrt(24–32+2·2·3–3)= sqrt(16–9+12–3)=4. This is an example in a calculus class, and it was chosen so that f(2,3) was nice.

Then ∂f/∂x=(1/2)sqrt(x4–y2+2xy–2)–1(4x3+2y) and ∂f/∂y=(1/2)sqrt(x4–y2+2xy–2)–1(–2y+2x). We can evaluate these derivatives at (2,3):
∂f/∂x(2,3)=(1/2)(1/4)(4·23+2·3)=(38)/8 and ∂f/∂y(2,3)=(1/2)(1/4)(–2·3+2·2)=–2/8.

If we want a linear approximation to f(2.03,2.98), then we may use the following formula:
f(2.03,2.98)≈f(2,3)+∂f/∂x(2,3)(.03)+∂f/∂y(2,3)(–.02).
Here the change in x from 2 to 2.03 means that h is .03 and the change in y from 3 to 2.98 means that k is –.02. The linearized approximation gives us 4+(38/8)(.03)+(–2/8)(–.02) which is 4.1475. The "true value" (up to 10 decimal places!) of f(2,3) is 4.147314409.
As I declared in class, in 1910 this would be nearly amazing! Accuracy to almost 4 decimal places. But, yuh'see, we have electronic devices to compute this. The idea of linear approximation is what will be important.

The spaceship in a nebula
My online dictionary states that a nebula is "a cloud of gas and dust, sometimes glowing and sometimes appearing as a dark silhouette against other glowing matter." So we could pilot a spaceship through a nebula. We might be concerned about the physical effects of the nebula, for example, the temperature. I'll assume that the spaceship measures temperature at the tip of its front. A point in the nebula will be located with rectangular coordinates, (x,y,z). The temperature at that point will be T(x,y,z). The rocket will fly a path so that at time t its location will be <x(t),y(t),z(t)>.
From this we can see that the temperature measured at the rocket at time t is T(t)=T(x(t),y(t),z(t)), and this is a composition. First we find out where the spaceship is at time t, and then we compute the temperature at that point.

Computing dT/dt
I would like to compute and understand the rate of change of the temperature, T, with respect to time. It turns out that this is a significant computation. Now T is a number and t is a number and T is a function (complication: there are intermediate variables x, y, and z) of t. So the derivative of T will involve taking its value at t+Δt and looking for the linearization multiplier. That is what we declared earlier. So here we go. I will try to accompany the steps of this somewhat elaborate manipulation with explanations.

T(x(t+Δt),y(t+Δt),z(t+Δt))=
                        We are "kicking" the time variable a little bit, and we would like to examine the change in the T variable.
T(x+x´(t)Δt+H.O.T.,y+y´(t)Δt+H.O.T.,z+z´(t)Δt+H.O.T.)=
  We use the fact that each of the components of the position vector are differentiable, so each function value at t+Δt can be replaced by the original value of the function, a multiplier (the derivative) which multiplies the disturbance, and higher order terms. If I were being careful, I would use different notation for each of the H.O.T.'s, but in practice people don't do that too often. You'll see why soon.
T(x,y,z)+(∂T/∂x)(x´(t)Δt+H.O.T.)+(∂T/∂y)(y´(t)Δt+H.O.T.)+(∂T/∂z)(z´(t)Δt+H.O.T.)+H.O.T.
 This is the differentiability of T. The changes in the inputs to T (there are three inputs) are passed outside. What happens? The linearization idea says that the changes are each multiplied by an appropriate partial derivative. And this isn't really exact (the numerical example showed this!) so there is also a H.O.T. from the differentiability of T. (Complicated? Sure is.)
T(x,y,z)+(∂T/∂x)(x´(t)Δt)+(∂T/∂y)(y´(t)Δt)+(∂T/∂z)(z´(t)Δt)+ (∂T/∂x)H.O.T.+ (∂T/∂y)H.O.T.+ (∂T/∂z)H.O.T.+ H.O.T.
 Now I did some multiplication and rearrangement. I pushed everything involving any of the Higher Order Terms to the end.
T(x,y,z)+(∂T/∂x)(x´(t)Δt)+(∂T/∂y)(y´(t)Δt)+(∂T/∂z)(z´(t)Δt)+H.O.T.
  Here is the important step, and it sort of asks you to think a bit about the ideas of calculus. All of the terms with H.O.T. are actually, all added together, just another big H.O.T.
T(x,y,z)+{(∂T/∂x)(x´(t))+(∂T/∂y)(y´(t))+(∂T/∂z)(z´(t))}Δt+H.O.T.
 This is the last rearrangement. Please, I hope you have the patience to see what has happened. We have the "old" unperturbed value of T(x(t),y(t),z(t)), and then we have a mess (not really, as you'll see) multiplying Δt, and then, finally, we have a whole bunch of things which logically are inside the H.O.T.

Now what? If f(v) is a function of one variable, then f(v+Δv)=f(v)+f´(v)Δv+H.O.T. identifies the derivative of f by what happens to small perturbations. The red stuff is the derivative, because it is the multiplier of the small perturbation of the input.

We carried out an elaborate analysis of how the temperature function changes. Now look at the result again:
T(x(t+Δt),y(t+Δt),z(t+Δt))=T(x,y,z)+{(∂T/∂x)(x´(t))+(∂T/∂y)(y´(t))+(∂T/∂z)(z´(t))}Δt+H.O.T.
The multiplier of Δt is dT/dt, so we see that

dT/dt=(∂T/∂x)(x´(t))+(∂T/∂y)(y´(t))+(∂T/∂z)(z´(t))

But this formula is not the point of the discussion. Look at the equation and think a bit. The luck and glory is recognizing that the mess on the righthand side is a dot product. In fact, look:

dT    / ∂T   ∂T    ∂T \    / dx   dy   dz \
-- =   --- , --- , ---  ·   -- , -- , --
dt    \ ∂x   ∂y    ∂z /    \ dt   dt   dt /  
                 Left                   Right
There are certainly many ways to organize this as a dot product, but this way turns out to give some insights that amaze me.

Right
The vector on the right-hand side is one we've looked at when discussing curves. It is the derivative of r(t), the position vector, so it is v(t), the velocity vector. This vector deals with the spaceship and its motion.

Left
This vector seems to be "new": it is the vector of all the first partial derivatives of T in order. This is called the gradient of T and is frequently written ∇T and is sometimes also called grad T. The upside-down triangle (or upside down Δ) is sometimes called "del" or "nabla". This vector can be computed only from the nebula information.

Thinking about the temperature derivative this way "decouples" (yeah, a word that's used) the influence of the nebula from that of the spaceship.

Now I will try to make a sequence of observations which might help people understand the excitement I feel thinking about gradient.

Observation 1
Let's imagine two spaceship trips through the nebula. Now these trips (voyages?) may be completely different except that at the time the two spaceships pass through the point (x,y,z), the spaceships have the same velocity vectors: that is, the spaceships are heading in the same direction and at the same speed. Their v(t)'s are the same. Then the rate of change of the temperature, dT/dt, that the two spaceships measure is exactly the same.

I asked students if they could deduce this from the physical and geometric aspects of the "scenario". I don't think I can. As a math fact goes, this is nearly obvious: since the v(t)'s are the same, the right-hand side doesn't change, and the nebula's temperature function is the same, so the left-hand vector ∇T doesn't change. Therefore the dot product, which computes dT/dt, is the same. But ... but ... what the heck ... can you "see" this physically? This is not the temperature at the point, but the rate of change of the temperature: the rate of change is the same if the velocity vectors are the same.

Observation 2
Now r´(t)=v(t), the velocity vector. Let me call the unit tangent vector u(t) here (in the discussion of curvature it was called T(t) but that will just be too darn confusing). Then v(t) is the same as (ds/dt)u(t) where ds/dt is the speed and u(t) is the unit tangent vector. In the formula ∇T·r´(t) the ds/dt effect just "filters out" of the dot product. If you travel twice as fast on the same path, then the rate of change of the temperature with respect to time is just doubled. So this is easy to understand. But the more subtle aspect is what happens as the direction changes.

Observation 3
Here I will suppose that ds/dt=1 for simplicity. Again, call the unit tangent vector, u, for unit vector. Then what can we say about ∇T·r´(t)? It is (ds/dt)∇T·u, or just (since I'm assuming unit speed) ∇T·u. But, hey, the dot product is also ||∇T|| ||u||cos(θ). This is ||∇T|| cos(θ) since u is a unit vector. Since cos(θ) is between –1 and +1, I now know that dT/dt is between –||∇T|| and +||∇T||.

How could we choose u so that dT/dt is largest? We need to make cos(θ) equal to +1. Therefore we need θ to be 0, and u should be a unit vector in the direction of ∇T. That is, choose u to be ∇T/||∇T||. To make the rate of change as much negative as possible, choose u to be –∇T/||∇T||, and then dT/dt will be –||∇T||.

An example (?)
Here is an example. If T(x,y,z)=x2eyz–5z3 then since ∇T=<∂T/∂x,∂T/∂y,∂T/∂z>, we compute:
∇T=<2xeyz–5z3,x2eyz–5z3(z),x2eyz–5z3(y–15z2)>
As far as I know this function and this computation has no great or special "meaning".

A better example (!)
Im my kitchen I have just finished baking my famous chocolate brownie pie and I left the oven door slightly open. Also I managed to forget to close the refrigerator. As a result, the contour lines of temperature (these are called isothermals) could like what is shown to the right. In what direction should I go (I am the little green man in the picture!) to most rapidly increase the temperature? In the direction of the gradient, which will point away from the isothermal curve that I am standing on. I will most rapidly decrease the temperature by traveling in the opposite direction, again directly away from the isothermal.

Observation 4
I could imagine that spaceship travels through the nebula on an isothermal surface. An isothermal is a collection of points where the temperature is all the same. We have seen this already: T(x,y,z)=C is a level surface (dimension 3) or level curve (dimension 2) or a contour {surface|curve}. But if the spaceship travels on such a surface, then the rate of change of the temperature must be 0. But then ∇T·v=0. This means that the velocity vector is perpendicular to the gradient. But then in turn this means that the gradient vector is perpendicular to the level surface, and it is perpendicular to the tangent plane of the level surface. In the kitchen, I would walk perpendicular to the contour lines to increase or decrease temperature most rapidly. I would walk along the contour lines if I wanted no rate of change of temperature.

Back to the example
Let me look more closely at the example with T(x,y,z)=x2eyz–5z3 when x=3 and y=2 and z=1. Then T(3,2,1)=9e–3. And ∇T=<2xeyz–5z3,x2eyz–5z3(z),x2eyz–5z3(y–15z2)> becomes ∇T(3,2,1)=<6e–3,9e–3(1),9e–3(–13)>=<6e–3,9e–3(z),–117e–3>

Now forget all that, and solve the following geometric problem:
What is the equation of a plane tangent to the surface x2eyz–5z3=9e–3 at the point (3,2,1)?
This could be, indeed, I claim, this is a hard problem. But if we now disobey my urging ("forget all that") I can tell you that ∇T(3,2,1) is perpendicular to the surface and to its tangent plane at (3,2,1). So I can write the answer, since I know a point and a normal vector to the plane requested:
    6e–3(x–3)+9e–3(y–2)+–117e–3(z–1)=0.
I think that solving such a problem so efficiently is really remarkable.

 
Topographic maps
A topographic map shows contour lines. Frequently while hiking people mind want to find the most direct route to the "top" (a mountain peak) or to the "bottom" (a creek?). They know by experience that the most direct route, only looking at the map, that is, only the geometry of the situation, would be to walk as nearly as possibly perpendicular to the contour lines.
This can be adapted into computational strategies for finding maxes and mins. If you can readily compute your function's gradient, then find maximums by going in the direction of the gradient. This is hill climbing. Find minimums by going opposite the direction of the gradient. This is the method of steepest descent. Of course these computational ideas don't always work, and there are many implementation details to worry about, but the general strategy is valuable.
Contour map example
So to the right is a made-up example of a contour map. The lines shown are at height multiplies of 100 feet. I hope that you can see (?) there is sort of one part of the map which deserves the name "peak" (in the upper left-hand corner). This map is a bit more complicated than the one I drew in class. I claim there is also a part of the map which could be named "the pits" (probably at least two of them!) and that's inside the two loops at the 300 foot height.
Hill climbing -- a way to get to the maximum
Now suppose we are at the point P on the map. How should we hike in order to get uphill as fast as possible? I claim that the directions shown will do that. Notice that I am not always walking towards "the peak", but I just want to always walk perpendicular to the contour lines -- in the direction of the gradient of the height function.
The method of steepest descent -- how to get to the minimum
This is a way people use to try to get to minima of functions in several variables. We start at the point Q and move in the direction opposite the gradient of the height function. In what I'm showing here, we take some sort of fixed step size (more sophisticated versions vary the step size) and after that step we keep going until we hit another contour line. Then we recompute the direction (minus the gradient) and step again. You can see, I hope, that this path is headed for one of "the pits".
These methods don't always work and don't always work efficiently, but they are easy to understand, they can be done in many dimensions, and the programs run fast.
The most change
I could also ask where there is the most change in height in a fixed distance. These are contour lines corresponding to equal changes in height. The heights will change most rapidly (as a function of the position on the map) when the contour lines are most closely spaced. This also corresponds, of course, to the region in the plane where ||∇ height|| is largest: the largest magnitude of the gradient vector. I think the region indicated (in magenta) is the region in this rather simple contour chart.

 

The QotD: tangent plane to an ellipsoid
Here's a neater example. Consider the ellipsoid (egg) x2+2y2+3z2=9. The point (2,1,1) is on this ellipsoid. What is the equation of a plane tangent to the ellipsoid at (2,1,1)? The gradient of the function x2+2y2+3z2 is <2x,4y,6z> and at (2,1,1) this is <4,4,6>. The equation of the tangent plane is 4(x–2)+4(y–1)+6(z–1)=0.

To the right is a Maple picture made by the commands which follow. I hope that the picture helps to convince you that the plane is the tangent plane.

A:=implicitplot3d(x^2+2*y^2+3*z^2=9,x=-5..5,y=-5..5,z=-5..5,grid=[20,20,20],
axes=normal,labels=[x,y,z],color=green,style=hidden);
B:=implicitplot3d(4*(x-2)+4*(y-1)+6*(z-1)=0,x=-5..5,y=-5..5,z=-5..5,axes=normal,
labels=[x,y,z],color=green,style=hidden);
display({A,B});

                        10/1/2010

On Thursday I did something in class that I've only done once before: I asked a student to leave.

The student had been repeatedly texting. I had remarked on this to the student several times. Students should not use electronic devices in class. This includes any messaging or recording devices. Students should try to be alert in class.

Our mutual task in meeting for lectures is to encourage learning. My part is to prepare and present relevant, useful material. Your part is to concentrate and participate. You can't do this while texting or listening to MP3's. You also can't do it if you are systematically tired.

                                               Sincerely, S. Greenfield


Tuesday, September 28, sections 12-14 and sections 15-17, lecture #9

Volunteers
A number of happy student volunteers found some partial derivatives.
  1. Mr. Selwyn Joy and Ms. Janice Wong and Mr. Jonathan Yeh and Mr. James Pak kindly found the first partial derivatives of arctan(y/x), which arises in steady-state heat distributions.
  2. Mr. Mark Jaeger and Ms. Diana Baginski and Mr. Philip Gallaro and Mr. Kazim Hassan searched and found the first partial derivatives of e-x2/t/sqrt(t) which is used to analyze one-dimensional diffusion processes.
    For your information, to the right is a picture of this function. x is position on the horizontal axis, and the height of the graph is the amount of the substance (diffusing from the origin) at time t. The picture (clearly!) loops. It was produced using Maple.
  3. Mr. Edward Lee and Mr. Ihsancan Turan and Ms. Laura Norkute and Ms. Rylan Uherek found the first partial derivatives of 1/sqrt{x2+y2+z2}, used as a potential (we will see later what that is) for gravitation and electromagnetism.
Slicing and partial derivatives
SWe define partial derivatives. Please realize that everything we are doing can be done in any number of variables (want a picture of 703 dimensions?) but I'll stick with 2 dimensions here because I can draw pictures and I like pictures.

limh→0(f(x+h,y)–f(x,y))/h=∂f/∂x
limk→0(f(x,y+k)–f(x,y))/k=∂f/∂y

I'll use h for little changes in the first variable and k for little changes in the second variable.

Now a picture. Look at a graph of z=f(x,y). We can slice this in various ways. For example, we could slice this by a plane perpendicular to the y axis with y fixed. This will give sort of an z-x curve. We could "lift" that curve up and just consider it as a function of one variable, x, and then look at the derivative. That's ∂f/∂x. This notation, with a sort of curly d, is not terrific, and it can be (as we will see) quite confusing. But it is what almost everyone uses. Similarly, we could slice by a plane perpendicular to the x axis with x fixed and consider the derivative of the resulting curve or function. That will be ∂f/∂y.

A betting game (?)
Consider this situation: suppose f(x,y)=(sin(y4)x–7)3. Then I flip a (fair) coin. If it lands "heads", I ∂/∂x this function. If the coin shows "tails", I ∂/∂y the function. What's going to happen? I asked students to speculate about this. Almost everything that students said was correct. I sometimes tried to distract people from the real question by making interesting and true assertions. For example, ∂100f/∂y100 yields a mess with 342 terms. And 200 y derivatives gives an algebraic mess with 680 terms. These computations were not done by hand, but with Maple. The expressions begin to swell (get larger and larger). I asked students if they thought that this sort of growth would be likely under the conditions of the experiment. Some students kept remarking about ∂/∂x and I kept "distracting" with facts about ∂/∂y. Here is an important and relevant result for this game.

Clairaut's Theorem (equality of "mixed" partial derivatives)
Suppose f(x,y) is a function of two variables, and the mixed partial derivatives fxy and fyx both exist and are both continuous. Then these mixed partial derivatives must be the same.

Certainly in Math 251, the hypotheses of the theorem will be satisfied. There are examples (similar in nature to the bizarre functions previously given) where things aren't the same. But in this course, the mixed partials will be continuous and therefore will agree. The verification of this result is in the textbook and uses the Mean Value Theorem of 1 variable calculus. I tried to use the analogy of row and column differences in a spread sheet to show that this result is believable.

 
O.k., here, graphically, is what I tried to discuss in class. If you are familiar with a spread sheet, you know there is usually a rectangular array of "cells". Specific cells can be "addressed" with letter/number combinations such as C5. Here I choose to label the addresses of the cells with (x,y). A vertical displacement (row change) is denoted by "+h" in the first number, and a horizontal displacement (column change) with a "+k" in the second number. The contents of the cell at (x,y) will be f(x,y). Then I asked students to imagine either a colum,n difference or a row difference, followed by the other difference. The results are as shown below. The column-followed-by-row result is (f(x+h,y+k)-f(x+h,y))-(f(x,y+k)-f(x,y)) The row-followed-by-column result is (f(x+h,y+k)-f(x,y+k))-(f(x+h,y)-f(x,y)).
These are the same! (Check the signs.)
Clairaut's Theorem is exactly this sort of manipulation, but some adjustments need to be made (divisions by h and k, limits, etc.) and these adjustments are handled by the 1 variable Mean Value Theorem. The details are in the textbook.


 

The result implies, for example, that if we look at the "crowd" of all the possible third partial derivatives of a function of two variables:
fxxx   fxxy   fxyx   fyxx   fxyy   fyxy   fyyx   fyyy
it may seem that there are eight possibilities. But due to Clairaut, there are only these four:
fxxx   fxxy   fxyy   fyyy
The effect of the "concentration" gets even stronger as the number of derivatives increases. For example, there could be 16=24 fourth order dervatives for a function of two variables, but Clairaut's Theorem applies to show that only 5 of them can be distinct.

Return to the game
How does Clairaut influence my original question? Again, I was perhaps not the most helpful person in leading the discussion, but eventually the most relevant fact appeared. The function f(x,y)=(sin(y4)x–7)3 is a cubic (degree 3) polynomial in x. That is, it can be written as
(Stuff0)x0+(Stuff1)x1(+Stuff2)x2+(Stuff3)x3
where each of the "Stuff" terms is some function only involving y. An x derivative, ∂/∂x, lowers the degree in x. And four x derivatives will leave us with 0. If you toss a coin a large number of times, it is overwhelmingly likely that there will be at least 4 heads, and therefore, in the differentiation choices, at least 4 x derivatives. So since we can reorder these mixed partials in any way we want, we could put those four derivatives first. And the result will be 0. So, almost surely, if we toss a coin many times, and follow the directed sequence of derivatives, the result will be 0.

What does differentiable mean in 1 variable?
What does f´(x)=Q mean? The definition we all tried to memorize (for a while, anyway) went something like this:

 
limw→0(f(x+w)–f(x))/w=Q
 

This definition is difficult to compute with because it has division and subtraction. It also has the lim notation. It really means that the equation is approximately true without lim: so if we allow some sort of error (which is small whant w is small) we can omit the lim.

People frequently "unroll" the limit statement to get this version:

 
f(x+w)=f(x)+Qw+Error
 

Here f(x) is the old, unperturbed output or response of f to the input of x. We perturb (kick?) the function box with a small w. The response of f to x+w can be decomposed (if f is differentiable!) into f(x), the old response, a linear or first-order disturbance, Qw, and "Error". The Error term is very complicated. Of course it will depend on f and x and w (and maybe the phase of the moon). But what is most important about the error term computationally is that it approaches 0 faster than first order. Frequently in applications the Error term is thought of or labeled, H.O.T. for "higher order terms". A function is differentiable in one variable exactly when its response to a small kick can be described as above. This corresponds geometrically to a well-known phenomenon that can be demonstrated nicely using graphing calculators. If you take a point on the graph of a differentiable function and zoom in repeatedly on the graph (centered at the point) within usually a few "zooms" the graph begins to look like a straight line (this is certainly not true of f(x)=|x| at x=0!). Therefore the graph of y=f(x) is (approximately) locally linear exactly when the function is differentiable. It is the property of being approximately locally linear which turns out to be important in higher dimensions. The derivative is the multiplier effect of a small change in the input.

Differentiable in two variables
The functions we want to consider are called differentiable and have exactly the property that they can be approximated nicely in a fashion similar to the 1 variable case.
f(x,y) is differentiable at (x,y) if there are numbers Constant1 and Constant2 so that for h and k small, f(x+h,y+k)=f(x,y)+Constant1h+Constant2h+Error, where the Error term→0 faster than |h|+|k| (so, faster than first order -- it is H.O.T.).

QotD
SInce i am concerned that people don't know what they should know about ordinary computation of derivatives, I wanted to give another QotD asking for such computation. Several students remarked to me after class that the example here is nearly evil.

If f(x,y)=xy, I asked for ∂f/∂x and ∂f/∂y and ∂2f/∂x∂.
I was very nice and gave the hint that the problem would be much easier if xy were written as eSOMETHING. After a while, I further hinted that x could be written as eSOMETHING ELSE and this might help. Well, x=eln(x) so that xy=(eln(x))y=eyln(x) (repeated exponentiation multiplies!). With this description, derivatives of f can be computed using the Chain Rule and Product Rule. You just need to gear your head, in each computation, to recognize what the variable is and, therefore, to treat everything else as a constant. Here we go:

The results can be made a bit prettier with some algebraic effort, but why bother?

Yes, it is really fair for me to ask such questions and expect that students can do such computations: these results are really straightforward.


Thursday, September 23, sections 12-14, and Friday, September 24, sections 15-17, lecture #8

Let's begin again and look at f(x,y)=x2+y2. A simple picture (using the Maple command plot3d) is shown to the right. Here both x and y range from –10 to 10. The result is a sort of cup: this surface is called a paraboloid.

Certainly this is not a complicated surface, but I want to discuss various options in looking at it. These options will be useful for more complicated examples.

I looked at this graph and studied curves for fixed values of x and y (called traces in the textbook). These are just (?) parabolas opening up. Piecing them together to get this parabolic cup is not totally obvious. To the right are some traces. Shown in green is the result of intersecting the surface with the plane x=–3. Shown in red is the result of intersecting the surface with the plane y=2. So this surface can be thought of as made of a collection of parabolas, assembled in either direction.
Another kind of plot, or, anyway, some geometric clue to the nature of the function, can be gotten by looking the contours of f(x,y). There are topographic maps (say, used by hikers) which give a two-dimensional representation of the information in the surface picture above. Pick a constant, C, and look at the (implicitly defined) "curve" f(x,y)=C. I put quotes around the word "curve" because maybe it doesn't have to be a neat nice curve. (An example was discussed in class, and is below.) To the right is a collection of contours for f(x,y)=x2+y2. These contours correspond to the positive integers 1, 2, 3, 4, 5, and 6. I asked students to sketch this graph for yesterday's QotD. There are some real subtleties in this graph.
Please notice how these contours, which are at evenly spaced "heights", get closer together as the three-dimensional graph gets steeper. Of course, if the contours are not labeled with the values of the constants, I can't tell if the function is increasing or decreasing! This picture was made with contourplot, another part of the Maple package, plots.

Reality?
I attempted to convince people that real pictures can be much more horrible. I displayed a historical relic -- a prinout of NMR data for a purified (and cooled) protein. What I showed could be thought of as contours for a strange function (simplified, the strength of magnetic response at various frequencies of ... oh heck, there's no way I'll attempt to explain NMR in 37 seconds or less. The history in what I displayed (the objects are about 20 or 25 years old) is that printouts are no longer made. The curves are displayed, and, in fact, various kinds of AI (artificial intelligence) algorithms are used to analyze these curves.

Many phenomena observed in "real life" are much more complicated than the baby examples we'll be analyzing in this course.

 
Here is one of the variations which can be produced with the plot3d command. It has the three-dimensional (the two-dimensional picture of the three-dimensional graph!) plot with the contour lines shwon at the correct places on the graph. There are all sorts of views which can be obtained using the options of plot and plot3d . Sometimes these variants can be useful.

When this picture is rotated so we are looking down from the positive z-axis, a view similar to the contour lines shown previously is obtained.
 

The origin in this paraboloid graph is a local and absolute minimum. I briefly sketched a graph of f(x,y)=–x2–y2, which is an upside-down version of what was just drawn. So it has a local and absolute minimum. That's not too new. But let's consider something weird and wonderful.

A saddle
Consider the function f(x,y)=x2y2 (I'm trying to exaggerate the minus sign typographically in this since that's the most interesting part). The traces are again "just" parabolas. But here, when, say, y=2 (a plane transversal or perpendicular to the y-axis), we get f(x,2)=x2–4. So the curve z=x2–4 is a parabola opening up. As we change the y's in this sort of slice, the bottom of the parabola moves down for big |y|.
What about the other traces, with x=a constant? If x=3, then f(3,y)=9–y2. In the plane perpendicular to the x-axis given by the equation x=3, the curve is z=9–y2. a parabola opening down. And by thinking we can see the top of these parabolas moves up when |x| is large.

It may be difficult for a novice to see how to put these curves together. To the right is a Maple graph of this function for –3≤x≤3 and –3≤y≤3. When you look at this on the computer (please try!) you can rotate it and magnify it, and things might become more clear.

The contour curves of this function are shown to the right.

Consider x2–y2=3, for example. One point on this curve is x=2 and y=1, so look at the curve which goes through the point (2,1). This curve is a hyperbola. The hyperbolas (each has two pieces) corresponding to positive values of the function open left and right. On the other hand, there is a family of hyperbolas opening up and down. These correspond to negative values of the function.

And there's a special number. If x2–y2=0, then, since x2–y2=(x+y)(x–y), the points on this "curve" correspond to the two straight lines x=y and x=–y. (The strange little box near (0,0) in the contour plot is because Maple thinks that very small +/– numbers which it samples should also be 0. I am sorry about that. It may not be obvious looking at the curvy surface above that there are two straight lines on the surface. But there indeed are.

The point at the origin has a new kind of behavior, not found in one variable calculus. In certain directions it is a maximum. In other directions it is a minimum. This sort of point will be called a saddle point (not too strange a name) or a minimax.

You could imagine that there will be a whole range of behaviors in three hundred variables!
 

 
This saddle point is stranger than you might think. There are a heck of a lot of straight lines on this surface, many more than you might think. For example, if x=7t+4 and y=–7t+5 and z=112t–16 (these are parametric equations for some sort of straight line!), then we actually get a value on the surface z=x2–y2 for every value of t. This isn't obvious. I could insert the formulas are check algebraically that (7t+4)2–(–7t+5)2 is the same as 112t–16. Or I could just show you a picture, to the right. After loading plots I typed
>A:=plot3d(x^2-y^2,x=-10..20,y=-10..20,axes=normal,color=green);
>B:=spacecurve(<7*t+3,-7*t+5,112*t-16>,t=-2..2,color=red,thickness=3); 
>display3d({A,B});
and I got a version of the picture displayed to the right. On the computer, you can rotate it and play with it, and sort of convince yourself that the straight line actually is on the graph. Sort of amazing.
Don't believe me! You try it.
 

Two half planes
I defined a piecewise function to exercise "our" intuition.

        ( y  if x>0
f(x,y)= (
        (2x if x≤0
First, this function does depend on y. So we computed:
f(3,4)=4, f(–3,4)=–6, f(3,–4)=–4, f(–3,–4)=–6.
We discussed the graph of z=f(x,y).

The behavior on the two sides of the yz-plane (where x=0) is different. More globally, the "rear" halfplane (where x<0) is a half of a plane whose equation is z=y. This equation gives a plane tilted at 45o to the y-axis. The front half, where x>0, was a plane which was tilted up as y increased. The graph that is shown is Maple's version. There are some real problems with how the program shows this graph. The jumps, which we investigated carefully, are connected because what Maple does is just connect dots. So it connects the jumps even when these are not part of the graph of the function (in 1 variable graphs, this "connecting" idea can be turned off, but I don't know how to turn it off in several variables). There are no vertical line segments in this graph! I strongly recommend that you try to graph this function yourself, and rotate the Maple plot systematically to understand what's going on.

 
How to define this function
Use the following, please:
     >f:=(x,y)–>piecewise(x>0,y,2*x);

This command tells Maple that if x>0, then the value of f(x,y) is y. If the statement is false, the value is 2x. Then I loaded the package plots and just used the commands plot3d and contourplot.

The contour "lines"
The contour lines are also not sketched too well. Most particularly, the contour "line" f(x,y)=0 is very peculiar. It actually consists of the y-axis together with the positive x-axis. Maple doesn't want to draw anything like that, so it actually omits a line segment in this T-shaped contour line. I tried various options with contourplot but I could not get the T contour (C=0) drawn correctly. The other contours are again for integer level sets. The level sets for C>0 are horizontal half lines in the first quadrant. The level sets fo C<0 have two pieces. One part is a horizontal half line in the fourth quadrant, and one part is a whole vertical line in the left halfplane. This may be hard to visualize. I urged people to try to educate their intuition. The left lines are closer together than the horizontal halflines. This is all very very peculiar, but the worst is to come.
 

The suicidal bug
Now comes some of the harder stuff. I asked people to imagine that some bugs were "walking" on the graph of z=f(x,y). The green bug, whose path is shown to the right, strolls along in a path which is roughly circular around the origin. This bug runs into trouble at any point on the positive y-axis, where there's a drop. It also has problems along the negative y-axis, where again there is a big difference in heights. This is a very small bug. I have tried to indicate this by a sort of light reddish color surrounding these half-lines. The blue bug walks from the right halfplane to the left halfplane. It is careful to cross only at the origin. The blue bug is totally safe, and never comes across any severe height differences. So I would like to discuss (and name [define], since it is a math course!) the differences the bugs encounter more precisely.

Continuity
BBB Almost all of the functions we will consider in this course will be continuous (in fact, much nicer than just continuous -- they will be smooth in a way that I'll explain later. But I should at least remark on what continuity means. The definition should not be too surprising.

A function f(x,y) is continuous at a point (a,b) in its domain if lim(x,y)→(a,b)f(x,y) exists and is equal to f(a,b).
Of course this doesn't help too much if we don't know what lim(x,y)→(a,b)f(x,y) means.

So I will discuss this very briefly.

Limits in one dimension
In one variable, limits are relatively simple. To define limx→af(x) we look at how x gets close to a from both sides. There are some standard pictures and standard examples of bad situations. Below are a few, to remind you.
Bad limiting behavior in dimension 1
A jump

(y=x+7 for x<3, and 2x otherwise.)

Many wiggles

(y=sin(1/x) for x positive, y=0 otherwise.)

Several variables
In several variables limiting behavior can be quite complex, much more than with one variable. I tried to give a few examples.

Many straight line limits exist
I asked students to consider the function
f(x,y)=xy/(x2+y2)
This is an algebraic formula which behaves is a strange fashion for (x,y) near (0,0). We could try some values, but we can also take advantage of the appearance of x2+y2. Almost always that's a signal to at least attempt to understand things in polar coordinates -- that is, to take advantage of circular symmetry.


Since x=r cos(θ) and y=r sin(θ), we know that x2+y2=r2 and xy=r2cos(θ)sin(θ). Therefore
f(x,y)=xy/(x2+y2)=cos(θ)sin(θ)
The value of f(x,y) only depends on the angular part of the polar coordinate representation of (x,y) and not at all on the radial component. The graph is made up of a bunch of half lines all parallel to the xy-plane, radiating out from the z-axis. These halflines, since cos(θ)sin(θ)=(1/2)sin(2θ), all have height between –1/2 and +1/2.
A Maple graph of the surface over the first quadrant (x>0 and y>0) is shown to the right. I also attempted, with the help of a stalwart student accomplice, to "draw" the surface kinetically. The student "volunteer" held one end of a bungee cord under some tension (both in the student and the cord!) while the calculus instructor held the other end and walked around the student. The calculus instructor raised and lowered the cord twice and the student was asked to keep the end of the cord at the same level as the instructor's end. Therefore along every angle a limit existed, but as the angle changed, the limits changed. There were infinitely many different limits possible along straight line approaches to (0,0).

 
Always 0 on a straight line approach
The final example of misbehavior was the following function:
        ( 1  if y=x2 and x>0
f(x,y)= (
        ( 0 otherwise
This function has only two values, 0 and 1. Certainly if you "walk" towards 0 on a straight line approach in the second, third, and fourth quadrants, the function values are all 0 and therefore the limit is 0. What's not so obvious perhaps is the behavior of the function on straight line approaches in the first quadrant.
Look at y=x. This line intersections y=x2 only at x=0 and x=1. So if we "walk" towards the origin on this line from some large x>0 considering the values of the function f(x,y), the function will be 0 at every point except x=1 where it will be 1. Certainly the limit as x→0 will exist, and it will be 0.
In fact, the limit exists on every straight line approach to (0,0), and the value of the limit is 0. But the real, two-variable limit should not exist, because the values of f(x,y) do not get close to 0 as (x,y)→(0,0).

If you want more precise definitions of limits, here they are.

Limits, 1 dimension
Here we've got a function of one variable, and we want to define and understand limx→af(x)=L. The actual definition, frequently stated but rarely stressed in calc 1 classes, is the following: (and, yes, the Greek letters ε and δ are almost always used)

Given any ε>0, there is some δ>0 so that if 0<|x–a|<δ, then |f(x)–L|<ε.

One way to possibly understand this is uses the model of a "function box" as I did in class: a box labeled "f" which has input and output. In this model, the ε is an output tolerance. We'd like our outputs to be within ε of the ideal output (for this problem) L. Then the limit definition states that there is some input tolerance, δ, which when applied to stuff going into the machine (only allowing inputs within δ of the initial input) then the output tolerance will be satisfied. The definition itself may be difficult to understand for several reasons. First, it is a complicated logical statement, Second, it provides no structure for computing or even estimating δ when actually given an ε. To me, this is a bit distressing. But some understanding of the input/output model and its approximation properties is fine right now.

Limits, 2 dimensions
We looked at several examples which were not continuous and did not have limits at (0,0). Let me show you the actual mathematical definition of lim<x,y>→<a,b>f(x,y)=L. It is very analogous to the 1 dimensional definition quoted above:

Given any ε>0, there is some δ>0 so that if 0<||<x,y>–<a,b>||<δ, then |f(x,y)–L|<ε.

Again the ε and δare output and input tolerances, respectively. The interesting feature to me is |<x,y>–<a,b>|. This means the distance from <x,y> to the point <a,b>. This is distance in any direction, along any path. The examples we saw last time only considered approaches to <a,b> along straight line segments. This turns out not to be enough. You've got to allow any paths, and, in fact, allow consideration of all points close to <a,b> (a sort of blob completely surrounding <a,b>). I think this makes limits much more "strict" in several dimensions.
 

QotD I introduced ∂ by just using it. This was an interesting pedagogical exercise.
Example 1 If F(a,b,c)=a2b–3bc3 then ∂F/∂a=2ab and ∂F/∂b=a2–3c3 and ∂F/∂c=–9bc2.

Example 2 If F(a,b,c)=a3sin(7b–5c2) then ∂F/∂a=3a2sin(7b–5c2) and ∂F/∂b=a3cos(7b–5c2) and ∂F/∂c=a3cos(7b–5c2)(–10c).

The QotD was to compute ∂F/∂a, ∂F/∂b, and ∂F/∂c if F(a,b,c)=a2e(7b–5c3).
So ∂F/∂a=2ae(7b–5c3), ∂F/∂b=a2e(7b–5c3)7, and ∂F/∂c=a2e(7b–5c3)(–15c2).

It turns out that ∂FUNCTION/∂VARIABLE, when FUNCTION depends on many variables, means that all variables except the variable specified are thought of as constants, and then the derivative of the function with respect to the remaining variable is computed with the help of the usual algorithms.


Tuesday, September 21, sections 12-14 and sections 15-17, lecture #7

Reminder
Tomorrow (Wednesday, September 22) only the recitations will meet in computer labs. Please remember the change in location for this meeting only. The purpose is to allow you to get some acquaintance with Maple.

Space curves and curvature
Now let's analyze a general space curve. If r(t)=x(t)i+y(t)j+z(t)k (the position vector), then r´(t)=x´(t)i+y´(t)j+z´(t)k=(ds/dt)T(t) is called the velocity vector. Again, T(t) is called the unit tangent vector and is a unit vector in the direction of r´(t). ds/dt is the speed, and is sqrt(x´(t)2+y´(t)2+z´(t)2), the length of r´(t). We use ds/dt also to convert derivatives with respect to t to derivatives with respect to s, as in two dimensions using the Chain Rule.

Since T(t)·T(t)=1 differentiation gives T´(t)·T(t)+T(t)·T´(t)=0. But the dot product is commutative, so this is 2T´(t)·T(t)=0 or just T´(t)·T(t)=0. This means that T´(t) and T(t) are always perpendicular. In fact, we are interested in dT/ds, which is the same as (1/(ds/dt))T´(t). It is usually easier to compute T´(t) directly, however, and "compensate" by multiplying by the factor 1/(ds/dt).
Think about this sentence, which states something used repeatedly in this course:

Any non-zero(!) vector is the product of its magnitude multiplied by a unit vector in its direction.
For dT/ds, the magnitude is defined to be the curvature, κ, and the unit vector is defined to be the unit normal N(t). This essentially coincides with what's done for plane curves, when curvature was defined to be dθ/ds.

Back to the right circular helix
Here r(t)=a cos(t)i+a sin(t)j+btk, the position vector. The velocity vector is r´(t)=–a sin(t)i+a cos(t)j+bk. The length of this velocity vector is sqrt([–a sin(t)]2+[a cos(t)]2+b2). This simplifies because we know that sin2+cos2=1. There are very few other curvature computations which are so simple.

So ds/dt=sqrt(a2+b2) (the length of the velocity vector, which is the speed) and we get the unit tangent vector by dividing the components of r´(t) by ds/dt. So T(t)=<1/sqrt(a2+b2))(–a sin(t)i+a cos(t)j +bk>. According to what we just did, if we differentiate this we should get a vector perpendicular to T(t). Here we go: d/dt(T(t))=(1/sqrt(a2+b2))(–a cos(t)i–a sin(t)j+0k). If you take the dot product of this with T(t) you will get 0 (the strange-looking signs make that true). But this is d/dt(T(t)) and, for curvature, we need d/ds(T(t)). The Chain Rule suggests that I divide d/dt(T(t)) by ds/dt to get d/ds(T(t)). If I do this, the result is {1/(a2+b2)}(–a cos(t)i–a sin(t)j+0k). Wow. We are not done yet. This should be κN: that is, it should be the curvature, a positive number, multiplying a unit vector, and this is the unit normal vector. If you stare at what we have you should eventually see the following:

dT/ds={1/(a2+b2)}(–a cos(t)i–a sin(t)j+0k)=[a/(a2+b2)](–cos(t)i–sin(t)j)
Here what is in blue is a scalar, multiplying what is in green which is a unit vector. Therefore what is in blue is κ, the curvature, and what is in green is the unit normal vector. By the way, this is why everyone (and me too, darn it, me too!) uses formulas such as those in the book to compute these things, because direct computation from the definition is too complicated.

If κ=[a/(a2+b2)] for the helix, how does this match up with our forecasts? Here is the scorecard.

Very few other computations of curvature are this simple.

To the right is another picture of the helix, "decorated" with one appropriate pair of T and N. The T is 1/sqrt(a2+b2))(–a sin(t)i+a cos(t)j+bk which is supposed to be a vector of length 1 tangent to the helix in the direction of motion. The N is –cos(t)i–sin(t)j. This is a vector with no k component, so it is horizontal, parallel to the xy-plane. Also, it points directly towards the axis of symmetry (the z-axis).

 
Here are some pictures of various helices produced by Maple (the plural of "helix" is "helices").
The pictures below were produced using the command
spacecurve([a*cos(t),a*sin(t),b*t],t=0..6*Pi,axes=normal,color=black,thickness=2,scaling=constrained,numpoints=180);
The procedure spacecurve is loaded as part of the plots package using the command with(plots);. I used the option scaling=constrained in order to "force" Maple to display the three curves with similar spacing on the axes. Otherwise the x and y variables would be much altered in each image. I hope that these pictures help you understand what curvature represents.
Some helices: x=a cos(t); y=a sin(t): z=bt.
a=100 & b=5
κ=.01
a=10 & b=10
κ=.05
a=5 & b=100
κ=.0005

The curvature of the twisted cubic
O.k., I tried to compute the curvature of r(t)=ti+t2j+t3k. I used a formula from section 13.4:
     κ=||r´(t)xr´´(t)||/(||r´(t)||)3.
Even though this formula looks weird, it is much better to use than trying to work through the definitions. I have tried using the definitions with this example, and the computations are terrible.

So r´(t)=1i+2tj+3t2k and r´´(t)=0i+2j+6tk. Now for the cross product computation:

   | i  j  k | 
det| 1 2t 3t2| = det|2t 3t2| i det|1 3t2| j + det|1 2t|k
   | 0  2 6t |      | 2 6t |       |0 6t |        |0  2|
And then I evaluated the 2-by-2 determinants, so we saw that the vector on the top of the formula for κ is (12t2–6t2)i–6tj+2k=(6t2)i–6tj+2k. But we need the magnitude or length of this for the top, and this is sqrt(36t4+36t2+4). Wow. The bottom is the three-halves (!) power of the length of the velocity vector, and this is (1+3t2+9t4)3/2.

Therefore (as they write in textbooks) the curvature of the twisted cubic is
     sqrt(36t4+36t2+4)
    ----------------------------
         (1+3t2+9t4)3/2

Now, after this {ludicrous|wonderful} computation, I do admit that I get almost nothing out of it. Here is this ridiculous formula, and what does it tell me? Maybe a little bit: If t is very large positive or very large negative, it seems to say that the curvature gets small (look at the "net" power of t on top, maybe a t2, and on the bottom "net" a sort of t6). I guess this means that the curve gets flatter as |t|→∞. Maybe this is interesting. (It sort of resembles y=x2 that way.)

 
How to think of all this?
What would I like "you" (especially the engineer and physics "yous") to take away? Another formula from the textbook (section 13.5) resembles exactly what we already saw in two dimensions:
     r´´(t)= (d2s/dt2)T + κ(ds/dt)2N
This is a decomposition of the acceleration vector into tangential and normal components, and it does have some interesting information about physical quantities as we saw in two dimensions for the straight line and the circle.
 

A little bit more ...
I hate to leave this subject since there is much more to be told, and everything turns out to be amazingly useful in a wide variety of applications. I hate to leave the subject, but the course is very dense. So: a little bit more. Think of an airplane flying along a curve in the sky (the dashed red line). Then the unit tangent and normal vectors are as shown. There's another vector, B, called the binormal vector, which is TxN. Since T and N both have length 1 and they are perpendicular, then B also has length 1 (sine of a right angle is 1). And B is perpendicular to both T and N.

The right-handed triple T, N, B is called the Frenet frame. Describing how this "frame" changes in time tells a lot about the curve. The curvature says how much the airplane is being pulled from a straight line. It measures how much the T is changing in the N direction. The green surface on the tail or the rudder of the plane is the major object determining κ. But the motion is three-dimensional. The way the binormal, B, changes, says how much the airplane is twisted out of the two-dimensional plane determined by T and N. The rate of change of B is called the torsion, and is determined mostly by the angles that the blue control surfaces have with the wings. When a plane takes off or is landing, these surfaces have relatively high angle to the wings. Let me do things with a bit more detail.

I apologize to any pilots, because what's above is not exactly correct. Actually, they would be thinking, "That's not correct at all". I am certainly simplifying. Reality is more complicated.
Also, right now I'm losing the advantage I got on the first day compared to the standard syllabus because I'm using an extra lecture for this material. Oh well.

How does B(t) change? Since B(t)·B(t)=1, differentiation results in 2B´(t)·B(t)=0, so B´(t) is orthogonal to B(t). But differentiation of B(t)=T(t)xN(T) results in B´(t)=T´(t)xN(t)+T(t)xN´(t). Since T´(t) is parallel to N(t) (because of our definition of N(t)!), the first product is 0 (another property of cross-product!) so that B´(t) is a cross-product of T(t) with something. Therefore B´(t) is also perpendicular to T(t). So B´(t) is perpendicular to both T(t) and B(t), and since only one direction is left, B´(t) must be a scalar multiple of N(t). The final important definition here for space curves is: dB/ds is a product of a scalar and N(t). The scalar is τ. That is supposed to be the Greek letter tau, and the minus sign is put there so that examples (the most important is coming up!) will work out better. This quantity is called torsion, and is a measure of "twisting", how much a curve twists out of a plane (the particular plane is the plane determined by T and N).

If a space curve does lie in a plane, and if everything is nice and continuous, then B will always point in one direction (there are only two choices for B, "up" and "down" relative to the plane, and by continuity only one will be used) so that the torsion is 0 since B doesn't change. The converse implication (not verified here!) is also true: if torsion is always 0, then the curve must lie in a plane!

 
Let me apply this to the right circular helix with a direct computation. I emphasize that this is by far the simplest example, and almost all curvature and torsion computations I've done in the last decade or so have used computer algebra systems.

So for our helix with r we found
    T(t)=1/sqrt(a2+b2))(–a sin(t)i+a cos(t)j+bk);  N(t)=–cos(t)i–sin(t)j;  ds/dt=sqrt(a2+b2).
So B(t)=T(t)×N(t). This is the scalar 1/sqrt(a2+b2)) multiplying the cross product of –a sin(t)i+a cos(t)j+bk and –cos(t)i–sin(t)j:

    |   i         j      k |
det |–a sin(t) a cos(t)  b | = b sin(t)i –(–b)(–cos(t))j+a(sin2+cos2)k
    | –cos(t)  –sin(t)   0 |
Yes, there really are 3 minus signs and a trig identity. So B(t) is the vector (1/sqrt(a2+b2))(b sin(t)i–b cos(t)j+ak). A few questions:
  1. Is B(t) a unit vector? Its length is (1/sqrt(a2+b2))sqrt(b2sin(t)2+b2cos(t)2+a2) which is (1/sqrt(a2+b2))sqrt(a2+b2)=1.
  2. Is B(t)⊥T(t)? The dot product of (1/sqrt(a2+b2))(b sin(t)i–b cos(t)j+ak) and 1/sqrt(a2+b2))(–a sin(t)i+a cos(t)j+bk) is ab–ab=0.
  3. Is B(t)⊥N(t)? The dot product of (1/sqrt(a2+b2))(b sin(t)i–b cos(t)j+ak) and N(t)=–cos(t)i–sin(t)j. is (–b)–(–b)=0.
Now for the torsion we need to find dB/ds. I compute dB/dt and get (1/sqrt(a2+b2))(b cos(t)i+b sin(t)j). To get dB/ds we multiply by 1/(ds/dt) which is 1/sqrt(a2+b2). The result is (1/(a2+b2))(b cos(t)i+b sin(t)j). This is supposed to be –τN(t) where N(t)=–cos(t)i–sin(t)j. Now notice that the darn minus signs cancel (that's one of the reasons for the minus in the definition of τ) and we see clearly that for a right circular helix, τ=b/(a2+b2). If b=0 this is a circle in the plane, and the torsion is 0. Intuition about torsion is hard to get.

Curvature tells how T changes, and torsion tells how B changes. What about N? In fact, if we look at the expression N=BxT and differentiate, using the product rules again, no new quantities are needed (this is done in detail below). The result is called the Frenet-Serret equations (also called the Darboux equations in mechanics):

dT/ds =  0 + κN +  0
dN/ds =–κT +  0 + τB 
dB/ds =  0  –τN +  0
These are three vector differential equations, so they actually represent nine (9) scalar differential equations and there is considerable significance in the way these equations look. If you want to move robot arms or analyze molecular backbones, then ... you may need to deal with these equations.

The first and third equations are used to define κ and τ. Their information content is that the rate of change of both T and B is in the direction of N which is maybe a bit surprising. The middle equation is the one that, to me, is really startling: the rate of change of N can be predicted from the quantities already defined -- nothing new needs to be considered.

Getting the middle equation
Since N=BxT, we can differentiate with the appropriate product rule, using the other two equations. If we d/ds, then dN/ds=dB/dsxT+BxdT/ds=–τNxT+BxκN. Remember that T,N,B is a right-handed coordinate system. Therefore TxN=B so that NxT=–B. And NxB=T so that BxN=–T. Therefore dN/ds=–τNxT+BxκN=τB–τT. This is the middle equation in the collection above.
 

κ and τ specify the shape of a curve
Reamrkably, just knowing curvature and torsion is enough information to get anything geometic about a curve. This is not at all obvious, and also, even though these quantities specify a curve, answering questions about the curve which might be interesting in applications (using just the curvature/torsion information) may not be simple in practice: the information is there, it is all there, but figuring out how to get it and getting it may be difficult. In connection with this idea (κ/τ determine everything, but perhaps not accessibly or simply) I told my favorite math joke:

JOKE
Several people are in a hot-air balloon, trying to land over a fog-shrouded countryside at the end of a long day. The balloon dips down low and they see the ground faintly. Spotting a person, one of them calls down: "Where are we?" Some minutes later the wind is carrying them away and they hear faintly, "You're in a balloon!" One person in the balloon gondola says thoughtfully to the other, "It's so nice to get help from a mathematician." The other says, "How do you know that was a mathematician?" The first replies, "There are three reasons: it took a long time to get the answer, it was totally correct, and, finally, it was absolutely useless."
 
Stay calm!
Consideration of torsion is not officially part of the course, and neither are the Frenet-Serret (Darboux) equations. And also no binormals. So stop worrying. There is more about all this in the textbook if you are interested. And also in books about motion, molecules, materials, ...

A formula for τ
Here is a formula for torsion from Wikipedia. While curvature depends on the first two derivatives, the torsion is more complicated and depends on the first three derivatives. I do not believe that sane people should memorize such formulas (unless they will work closely with them over a long period of time).

If r´, r´´, and r´´´ are the first three derivatives of the position vector (velocity, acceleration, and what is sometimes called jerk), then

    (r´×r´´)·r´´´   
τ= ---------------
     ||r´×r´´||2

 

New things ...
We move on to one of the major topics of the course. The word "several" is technical in mathematics, and means "more than 1". So we will study functions whose domain is several real variables, and whose range is inside the real numbers.

Warning
The basic definitions for calculus (limits and continuity) are much trickier when the domain has dimension >1. Some very strange things occur, and you should stay alert, please.

We'll begin with an almost ludicrously simple function: f(x,y)=x2+y2.

x2+y2
Here f(x,y)=x2+y2. This is a function defined by a formula (essentially all of the functions we'll consider in this course will be defined by formulas). The notation means that the input to the function is an ordered pair of numbers, (x,y), and the output is one number. Here the output for the ordered pair (–2,3) is 13.

 
Formalities: domain and range
The domain will be the collection (the "set") of all possible inputs. Just as in calc 1, if the function is defined by a formula, then the domain in this course will be all inputs for which the function makes sense. The usual restrictions that will concern us are:
  • Don't divide by 0.
  • Only square roots of non-negative numbers are allowed (same for other even roots).
  • Logarithms only for positive numbers.
These should all be familiar to you.

The range will be the collection of all possible outputs. You may remember from calculus that while determining precise domains is often possible but tedious, precise descriptions of ranges can be quite difficult (this can involve exact determinations of max and min values).
Here are some examples chosen to illustrate the likely possible restrictions. I certainly did not have time to discuss all of these in class.

f(x,y)=x2+y2
Domain
I think all pairs (x,y) of real numbers, all of R2.
Range
Since squares are non-negative, certainly the values of this function are non-negative. And f(0,0)=0, and f(sqrt(A),0)=A for A positive. I am just verifying precisely that the range is all non-negative real numbers.
f(x,y)=1/(y–x2)
Domain
So this example is chosen to illustrate the restriction about not dividing by 0. The domain is all pairs (x,y) of real numbers for which y is not equal to x2. Geometrically, this means all points of R2 which are not on the parabola y=x2.
Range
0 isn't in the range (it isn't the reciprocal of any number). But everything else is: check this by just looking at what happens to (0,A), which gives 1/A for all non-zero A's.
f(x,y)=sqrt(y–x2)
Domain
So this example is chosen to illustrate the restriction about square roots. The parabola y=x2 divides R2 into two pieces. One piece contains, say, the point (3,4) ("below" the parabola). This point has y–x2=4–32=–5<0, so (3,4) is not in the domain of this function. The domain is the "other" piece of R2 and also those points which are on the curve y=x2.
Range
The range is all non-negative numbers. Again, to check this you could look at what happens to (0,A) for A≥0.
f(x,y)=ln(y–x2)
Domain
I still must "throw out" the part of R2 which is below the parabola. But here inputs to ln must be positive, so the domain does not include the curve y=x2. The domain is all of the points in R2 which are above the parabola.
Range
The range is the range of ln, which is all real numbers.

 


Kinds of graphs
Let me return to the simplest of the functions I just considered: f(x,y)=x2+y2. There are various graphs which are commonly used. Maybe the simplest is to consider the points (x,y,z) in R3 which satisfy the equation z=x2+y2: this is usually called the graph of the function. A Maple representation of this graph is shown to the right, and the procedure which produced it is plot3d, part of the plots package. This is rather a simple function, and I hope you can see the shape of this surface. It is a cup, axially symmetric around the z-axis. It is called a paraboloid.

Contours
A contour curve or just a contour of f(x,y) is the curve in the (x,y) plane defined by f(x,y)=c (where c is a constant). For f(x,y)=x2+y2 I sketched f(x,y)=1, a circle of radius 1 in the xy plane.

QotD
I aksed people to sketch the contour curves for this function (f(x,y)=x2+y2) and these c values: 0, 1, 2, 3, 4, 5.
To the right is a Maple plot using the countorplot command (there are actually 6 contours, -- one is a dot at the origin). What's subtle about this (at least to me) is that although the contour values are evenly spaced (0, 1, 2, 3, 4, 5) the spacing between the curves is not the same.
This is not completely trivial!


Thursday, September 16, sections 12-14, and Friday, September 17, sections 15-17, lecture #6

Reminder
Next Wednesday (September 22) the recitations will meet in computer labs. Please remember the change in location for this meeting only. The purpose is to allow you to get some acquaintance with Maple.

Today we'll deal with a topic which has new ideas and new computations. Some of my discussion will be different from what's in the text.

The dynamics of a moving particle in space is one of the triumphs of basic vector calculus. Some effort is needed to understand the conclusions because even the motion of just one particle can be complicated. There are kinetic aspects, having to do with the specific motion of the particle (its parameterization) and also geometric aspects, where properties of the shape of the path are analyzed.

The geometry of space curves, curves in R3, as seen from the point of view of calculus (called "differential geometry of space curves") is a subject which originated in the 1800's, with some ideas coming from Euler as early as 1750. The material presented here was stated in about 1850-1870. When I learned about this material in college, it mostly seemed rather abstract and useless -- stuffy complicated formulas no intelligent person would care about. Like a numerous other judgments that I've made in life, this turned out to be completely wrong. The geometry of curves has within the last few decades become very useful in many applications: robotics, material science (structure of fibers), and biochemistry (the geometry involving the structure of big molecules such as DNA), and computer graphics: amazingly useful formulas and ideas!

We need from last time the idea that arc length is (theoretically, at least!) gotten by integrating the speed with respect to time. That is, using the letter s to represent arc length, if s(t) is the length of a curve from t0 to t, then s(t)=∫t0t||v(t)|| dt. We can rarely compute s(t) exactly. This won't be a major problem, since we will actually need a related version of this equation, in differentiated form: ds/dt=||v(t)||. This is an application of FTC (the Fundamental Theorem of Calculus). We will use this to switch around from t derivatives to s derivatives using the Chain Rule:
(dFROG/ds)(ds/dt)=dFROG/dt if FROG is any function of t.

Why might we prefer the variable s to t?

Our major goal in today's lecture is to analyze the bending of curves, and define and compute a quantity called curvature, which measures how a curve bends. How can we initially imagine curvature? Well, we could think about driving a car along a winding road. If the road bends abruptly, we might think it should curve more -- that is, the curvature should be bigger. But if you think about your experience driving, you really observe a sort of "apparent bending". You could imagine driving along a twisty road at twice the speed, and then the road would seem to bend twice as fast. Some of the bending effect involves the speed at which the road is traveled, and some of it is somehow intrinsic to the road. We will try to get an intrinsic notion of bending by imagining we are traveling at unit speed on the road -- that is, by using the rate of change with the arc length s as the parameter. This is weird because we can't generally compute s. But we can compute rates of change with respect to s using the Chain Rule equation mentioned above. Let me begin with two dimensional curves, called plane curves, and then go on to the three dimensional version, space curves.

Before actually getting our hands dirty, let me mention some special test cases which will allow us to check our results when we're done.

Two-dimensional examples

#1: a straight line
A straight line does NOT bend, so it should have curvature 0.

#2: a circle
A circle should have constant curvature, since each little piece of a circle of radius R>0 is congruent to each other little piece, and, in fact, the curvature should get large when R gets small (R is positive), and should get small when R gets large (and looks more like a line locally).

#3: "the" parabola
Even y=x2 might be a good test to keep in mind, since there the curvature should be an even (symmetric with respect to the y-axis) function of x, and should be bell-shaped, with max at 0 and limits 0 as x goes to ±–∞. The curve actually bends most near the origin, and far away, when x is large positive or large negative, even though the curve is very steep, it gets really flat.

Now let's consider a plane curve, as shown. The position vector is r(t), and the velocity vector is r´(t), which has components dx/dt (or x´(t)) and dy/dt (or dy/dt). These components determine the angle θ between the velocity vector and the positive x-axis. You could think of θ as the angular deflection from straight ahead of a steering wheel as you drive along a road in the shape of the curve. Let's look at the rate of change of θ.

Since θ=arctan(y´/x´), we can find dθ/dt: it is (y´´x´–x´´y´)/(x´2+y2)2. This uses the formula for the derivative of arctan, the Chain Rule, and the quotient rule. I did this in detail in class. But we are really interested in the geometric information involved in θ's rate of change, so we want dθ/ds. To get dθ/ds we need to use a version of the FROG equation: dθ/ds=(dθ/dt)/(ds/dt)=(dθ/dt)/(x´2+y´2)1/2. The result is then called the curvature and the Greek letter κ (kappa) is used:

κ=dθ/ds=(y´´x´–x´´y´)/(x´2+y´2)3/2, a complicated formula.

Back to the test cases
The straight line
We compute for some constants A and B and C and D:
x(t)=At+B x´(t)=A x´´(t)=0 y(t)=Ct+D
y´(t)=C y´´(t)=0
so that the top of the curvature formula is 0.
κ=0 for any straight line.
A circle
Suppose we have a circle of radius R centered at the origin. Then parametric equations are not too difficult to get:
x(t)=Rcos(t) x´(t)=–Rsin(t)
x´´(t)=–Rcos(t) y(t)=Rsin(t) y´(t)=Rcos(t)
y´´(t)=–Rsin(t)
Let me look first at the bottom of the curvature formula. This is (x´2+y´2)3/2. Notice that x´2+y´2=R2([–sin(t)]2+[cos(t)]2)=R2, so the three-half (3/2) power is R3. The top is y´´x´–x´´y´ and this is –Rcos(t)(–Rsin(t))––Rcos(t)Rcos(t)=R2·1. The whole curvature formula is then R2/R3 which is 1/R.

When R is large, this is near 0. When R is close to 0 and positive, this is large positive. And the curvature is constant which it should be since the circle has the same local geometry at every point.
 

 
This will be the QotD for students to compute!

A parabola
Here I want to look at y=x2. A simple parameterization is good. So let's try:

x(t)=t x´(t)=1
x´´(t)=0 y(t)=t2 y´(t)=2t
y´´(t)=2
Now the bottom of κ, (x´2+y´2)3/2, becomes (1+4t2)3/2, and the top, y´´x´–x´´y´, is just 2. So κ=2/(1+4t2)3/2.

Here the "local geometry" definitely changes from point-to-point. The most curvy (?) part of the graph is at the origin. As x→±∞, although the graph gets steeper and steeper, the curve locally actually gets more and more flat: the curvature→0.
 


The smaller figure is supposed to be the curvature of the parabla.

Some physical consequences
Let's define the unit tangent vector, T, to be a unit vector in the direction of r´(t), the velocity vector. If θ is the angle that r´(t) makes with the positive x-axis, then T must be cos(θ)i+sin(θ)j (because we are in a triangle whose hypotenuse has length 1). Also r´(t)=(ds/dt)T, where ds/dt, the speed, is the length of r´(t). Now differentiate the formula for r´(t) using one of the product rules stated earlier.

We get r´´(t)=(d2s/dt2)T+(ds/dt)dT/dt. Since T is always a unit vector, its length is 1 so T·T=1 (length squared!). Differentiate this equation and use the product rule for dot products. The result is dT/dt·T+T·dT/dt=0. But scalar product is commutative, so the two terms are the same, and we can divide by 2 and get T·dT/dt=0. That means dT/dt is perpendicular to T. Some people claim to see this "easily" using physical thinking. I can't think physically that well, so I am not convinced.

But T is cos(θ)i+sin(θ)j, and dT/dt=(–sin(θ)(dθ/dt)i+(cos(θ)(dθ/dt)j= (dθ/dt)[(–sin(θ)i+(cos(θ)j]. Since dθ/dt=(dθ/ds)(ds/dt)=κ(ds/dt) we can rewrite dT/dt as κ(ds/dt)[–sin(θ)i+(cos(θ)j]. The term [–sin(θ)i+(cos(θ)j] is a unit vector perpendicular to T and is usually called the unit normal vector and written N. The equation r´´(t)=(d2s/dt2)T+(ds/dt)dT/dt turns into
    r´´(t) = (d2s/dt2)T + κ(ds/dt)2N.

We have decomposed acceleration into the normal and tangential directions.
This can be significant physically and can help to understand physical and geometric situations. Here are two "extreme" cases. Realize, as you consider them, that since F=ma and m, mass, is a positive scalar, force is zero exactly when acceleration is zero and also that force and acceleration have the same direction since multiplication by a positive scalar doesn't change direction.

Motion in a straight line
Imagine a ball bearing (?) moving in a straight tube. I claim that the tube never "feels" any force from the walls of the tube. Why? For a straight line κ=0 always, the coefficient of the normal component of the acceleration, κ(ds/dt)2, is always 0, for any motion, and therefore, since force is a scalar multiple of acceleration, the force also must have zero normal component. So the ball bearing can be pushed along or back in the tube, but it is never pushed against the wall if you insist that the ball bearing move in a straight path. There is never any transverse force.

Motion in a circular arc
Now a more complicated thought experiment. Imagine the ball bearing constrained to move in a piece of a circular tube. All we know is that the object is moving, and its motion is part of a circular arc. I claim that then, no matter what, the ball bearing is "feeling" a transversal (normal) force from the "walls" of the tube. Why? The normal component of the acceleration (a scalar multiple of the force acting on the object) is κ(ds/dt)2. κ is not 0 since it is 1/(radius of the circle). And, since the object is moving, we also know that ds/dt is not 0. Therefore the product isn't 0, and the normal component of the acceleration isn't 0. There is always a normal force.

This is, of course, related to the wonderful primitive experiment of quickly spinning a bucket of water on a rope -- and the water is "pushed" into the bucket by the "centrifugal force". Hey, that force is a scalar multiple of the normal component of the acceleration of the bucket's motion. It really works.

The right circular helix
This will be our 3-dimensional "test case". We'll consider a right circular helix.

x(t)=a cos(t)
y(t)=a sin(t)
z(t)=b t
The quantities a and b are supposed to be positive real numbers. This helix has the z-axis as axis of symmetry. It lies "above" the circle with radius a and center (0,0) in the (x,y)-plane. The distance between two loops of the helix is 2π b. The "b" changes the pitch or angle of the screw threads modeled by the helix.

How should curvature behave for the helix? We discussed this and decided κ should have these properties:

Tuesday, September 14, sections 12-14 and sections 15-17, lecture #5

Limits, etc.
To do calculus officially we will need to have some notion of limits (for derivatives and integrals). When functions have domain in R and range in Rn (o.k., here just n=2 or n=3, but the same thing works for other n's), the theory and simple computations follow closely what happens with one variable. Let me explain.

Suppose r(t)=<a(t),b(t),c(t)> and we want to know about limt→Tr(t). That is, we wish to decide if limt→Tr(t) exists and is equal to a vector L=<A,B,C>. Well, we surely will consider what happens when |t–T| is small (but, please, restricting our attention to 0<|t–T| because otherwise we'll get into trouble when we look at derivatives because we would be dividing by 0). To have the limit exist, we would want ||r(t)–L|| to be small. But what is ||r(t)–L||? It is the square root of (a(t)–A)2+(b(t)–B)2+(c(t)–C)2. Notice that this is a sum of squares, and there is never any cancellation! The only way the sum can be small is if all of the pieces individually get small. This is sort of a "limits in parallel" statement:

Vector limits are the same as several simultaneous scalar limits.
Verifying limt→Tr(t)=L (where r(t)=<a(t),b(t),c(t)> and L=<A,B,C>) is exactly the same as verifying all of these statements:
(i) limt→Ta(t)=A; (ii) limt→Tb(t)=B; (iii) limt→Tc(t)=C.
Because of this, I won't bother doing a bunch of examples because they would be just running calc 1 examples in duplicate or triplicate (limt→0<t2+4,sin(5t)/t> is <4,5>).

Warning Things will not be this easy in a week when we interchange the dimensions of the domain and range!

The derivative
If r(t)=<a(t),b(t),c(t)> is a vector function of t, we can consider (1/h){r(t+h)–r(t)}. Inside the {} is a difference of vectors, and that's therefore a vector. Then multiplying by 1/h: that's a scalar multiple. So the result is a vector. It may happen that the limit as h→0 of this quotient may or may not exist. If it does, we'll say that r(t) is differentiable and that the limit, which is labeled r´(t), will be called the derivative. It is also called the velocity vector.

It doesn't take many examples to convince yourself that the (vector) derivative will exist exactly when all of the scalar functions a(t) and b(t) and c(t) are differentiable, and that the components of the resulting vector derivative will be the derivatives of these functions. That is, if a(t) and b(t) and c(t) are differentiable, then the vector function will also be differentiable, and r´(t)=<a´(t),b´(t),c´(t)>. Computing the derivative of such a vector function when the component functions are defined by familiar formulas involves nothing essentially new. I did a few examples.

Meaning of the derivative
The computations will generally not be interesting or new. The meaning of the quantities computed turns out to be significant and interesting.

The derivative balances out a large scalar, 1/h (when h is small), with a very small secant vector, r(t+h)–r(t). To the right is shown a possible picture of the situation. The the secant vector, r(t+h)–r(t) is in magenta. It is close to the curve, with its head and tail at two nearby points on the curve. That the limit exists is rather neat fact, that somehow the shrinking of the vector stabilizes with the increasing of the scalar amount, and that the direction tends to a fixed direction -- this is not obvious, and one should really not expect it! This direction is a vector, drawn in red. It is called the velocity vector or the tangent vector.

The magnitude: speed
The magnitude of the velocity vector, ||r´(t)||, is called the speed.

Velocity vector, tangent vector
I tried to argue, looking at a local picture of the path of a particle, that the direction of the velocity vector is tangent to the path of the particle. So the velocity vector is a vector tangent to the path.

 
So I had considered parameterizations of various portions of a straight line in the previous lecture. Let's look at the speed of the parameterizations.

<t,t,t> has derivative <1,1,1> with speed therefore sqrt(3): a constant. So this is a uniform speed parameterization.

<t3,t3,t3> has derivative <3t2,3t2,3t2> with speed therefore sqrt(3)3t2. For t large negative or positive, the speed is large. It decreases near 0 and the speed is 0 at 0. This is what we observed when considering the graph. Also the direction is always (except t=0!) <positive,positive,positive,>.

Now consider <t2,t2,t2>. The velocity vector is <2t,2t,2t>. For t<0 it points towards the origin, and for t>0 it points away from the origin, as we saw earlier. And the speed is sqrt(3)2|t| (remember that the square root of t2 is |t|, not t!). So this is large when |t| is large.

And finally <sin(t),sin(t),sin(t)> has speed sqrt(3)|cos(t)|, which varies periodically also as predicted.
 

Tangent line to a helix
Let's remember the helix from the previous QotD: r(t)=<5cos(2t),3sin(2t),8t>. What is this?
The first two variables describe uniform circular motion. The radius of the circle is 5, and that's the distance from the central axis of this curve, which turns out to be a helix. The 2 changes the angular velocity of the curve, and doubles it. The 8 affects the "pitch", the angle of the helix, and also the distance between "loops". When 2t changes by 2Π, the curve passes around one loop. That means the change in t is Π, so the change in z is 8Π.

Let's find the parametric equations of a line tangent to this helix when t=Π/3. The line must pass through r(Π/3)=<5cos(2[Π/3]),5sin(2[Π/3]),8[Π/3]>=< –5/2,(5/2)sqrt(2),(8/3)Π>. A vector in the tangent direction can be gotten from the velocity vector. So: r´(t)=<–10sin(2t),10cos(2t),8> which, when t=Π/3, gives <–5sqrt(3),–5,8>. Therefore the parametric equations for the line are:
x=–(5/2)–5sqrt(3)t
y=(5/2)sqrt(3)–5t
z=8t+(8/3)Π
To the right is a picture of both the helix and the tangent line just specified.

QotD
Suppose r(t)=<5t,3e(t2),tcos(5t)–2>.

  1. Find r´(t)
    Answer r'(t)=<5,6te(t2),cos(5t)–5tsin(5t)>.
  2. Find r(0) and r´(0).
    Answer r(0)=<5·0,3e(02),0cos(0)–2>=<0,3,–2> and r´(0)=<5,(6·0)e(02),cos(0)–(5·0)sin(0)>=<5,0,1>.
  3. Write parametric equations for the line tangent to this curve when t=0.
    Answer x=0+5t
    y=3+0t
    z=–2+1t.

Various formulas, especially product formulas

Here we suppose that v1(t)=<x1(t),y1(t),z1(t)> and v2(t)=<x2(t),y2(t),z2(t)> are both differentiable (this is logically the same as asking that all 6 of the scalar functions x1(t), ... ,z2(t) be differentiable.)

These formulas can be verified by writing out the components and using calc 1 techniques. For example, the dot product v1(t)·v2(t) is, in more detail, x1(t)x2(t)+y1(t)y2(t)+z1(t)z2(t), a collection of standard real-number products connected by sums. If we differentiate, the result using the standard product formula (I'll just write the result of differentiating the first term) is x1´(t)x2(t)+x1(t)x2´(t). I hope you see that this is the first part of each of the pieces in the sum which was given: v1´(t)·v2(t)+v1(t)·v2´(t).

I want to use calculus as a way of investigating the geometric properties of curves. This is a bit difficult because calculus applies to the parameterizations of the curves, and since there can be many sometimes wildly different parameterizations of the same curve, we'll need to be clever. This material was mostly developed between 1750 and 1850. It has been found useful recently in such subjects as molecular biology (describing how long molecules might twist) and computer graphics (what curves should shadow and light make?).

Arc length
If speed is the magnitude of the velocity vector, then since distance=rate·time, then, with variable speed, we need to chop up the time interval and compute and add up pieces of distance. This is the basic idea behind the definite integral, so it makes sense to compute the distance along a curve from time t1 to time t2 by this: ∫t1t2||v(t)|| dt.
O.k., this is good (all math is good!) but maybe some examples ... will show how silly it is.

Textbook example
Here is an example taken from a textbook. Suppose r(t)=<12t,8t3/2,3t2>. What is the length of the curve from t=0 to t=1?
This is a typical artificial problem in a textbook. We know that r´(t)=<12,8(3/2)t1/2,6t> so that the speed is ||r´(t)||=sqrt{122+122t+36t2}. Now we "remember" that distance=rate·time, and that the magnitude of the velocity vector is ds/dt, the speed. So the total distance traveled is ∫01sqrt{144+144t+36t2} dt. Now look at the coincidences. We can pull out the 36 from under the square root, and the result is 6∫01sqrt{4+4t+t2} dt, and, what a coincidence, 4+4t+t2 is (2+t)2 and the integral becomes 6∫01(2+t) dt=6(2t+(1/2)t2]01 and this is 6[2+(1/2)].

More realistically ...
Almost every combination of two or three functions, even rather "simple" functions that you try to use as a position vector, will yield something that can't be antidifferentiated in terms of familiar functions. For example, look at the following mess which is an attempt to compute the arc length of a fairly simple (?) curve:

> a:=t->t^3;
                                            3
                                 a := t -> t

> b:=t->cos(t);
                                   b := cos

> c:=t->exp(t);
                                   c := exp

> int(sqrt(diff(a(t),t)^2+diff(b(t),t)^2+diff(c(t),t)^2),t=0..1);
bytes used=1000188, alloc=917336, time=0.33
bytes used=2000404, alloc=1572576, time=0.63
bytes used=3007164, alloc=1703624, time=0.86
bytes used=4007360, alloc=2227816, time=1.11
bytes used=5007652, alloc=2620960, time=1.44
bytes used=6007984, alloc=2817532, time=1.79
                        1
                       /
                      |       4         2         2 1/2
                      |   (9 t  + sin(t)  + exp(t) )    dt
                      |
                     /
                       0
And that means Maple tried but can't figure out anything useful to reply except to echo the integral back to the questioner. My next step is
> evalf(%);
                              2.11113769
which asks for an approximate (numerical) evaluation of the integral. That's almost always what's going to be necessary. Oh well: the situation in textbook problems is much too nice. (By the way, a correction was made here on Monday afternoon, 9/20/2010: I thank Professor Gourevitch for noticing that I had not computed an arc length in what was written here before.)


Thursday, September 9, sections 12-14, and Friday, September 10, sections 15-17, lecture #4

As of Friday, September 10, the lectures have covered up to and including section 13.1. Please do related textbook problems.

Last time we learned how to specify a plane with two "chunks" of geometric data, a point on the plane, say (a,b,c), and a vector normal to the plane, say n=<d,e,f>. The result was (d)(x–a)+(e)(y–b)+(f)(z–c)=0. I emphasized that "simplifying" or doing a bunch of arithmetic is not necessary. So the plane 5(x–2)–2(y+4)+11(z–6)=0 is a fine equation. If you must, you can rewrite it as 5x–2y+11z=84 (and I hope that 84 is correct!).
We can reverse things a bit:

Reversing ...
6x+–3y+7z=20 is the equation of a plane.
What is a vector normal to this plane? If you followed the previous discussion, I hope you can see that the components of such a vector are the coefficients of x and y and z in order. So n=<6,–3,7> is a vector normal to this plane.
What are all (non-zero) vectors normal to this plane? These vectors are the non-zero scalar multiples of <6,–3,7>.
What are the coordinates of a point on the plane 6x–3y+7z=20?
Part of the difficulty in answering this question is that there are so many possible answers! For example, (0,0,20/7) is one answer, and so is (1,–7/3,1), and so is .... uhhh ... (1,000, –1,000, –8,980/7). There are many points on the plane.

There are many "exercises" which can be done with lines and planes. I did two in class, and a few more (not done in class) are shown with a strange background color.

 
Three points determine a plane
Again, Euclid declared that 3 points in space should determine a plane. So can we find an algebraic description of a plane through the points p=(3,3,–9) and q=(–5,2,1) and r=(4,2,2)? We need to find a vector n normal to the plane. The vectors pq and (say) qr are in the plane's direction. The cross-product of these two vectors will be perpendicular to the plane. So: pq=<–8,–1,4> and qr=<9,0,1>, and
   | i  j k |
det|–8 –1 4 |=–i–(–8–36)j–(–9)k=–i+44j+9k 
   | 9  0 1 |
is a normal vector. So –1(x–3)+44(y–3)+9(z–(–9))=0 is an equation of this plane. (We would get equivalent equations if we used q or r instead of p, or if we computed with pqxpr instead of pqxqr.)

Supposedly (several thousand years olds!) "3 points in space should determine a plane" but they may not always, because all 3 could lie on one line. What then would happen to our vector algebra? Then the two vectors we computed above would be scalar multiplies of one another and the cross product which resulted would be 0. So we would not be able to go on.
 

Exercise #1: the distance of a point to a plane
Suppose the point q is not on a plane, P. How can we find the distance from q to P? I did this in a rather clumsy way in class, mostly because I wanted to practice how to use line and plane equations. Let me start with specific information:
Suppose q=(2,1,3) and P is the plane specified by the equation 5x–7y+6z=10.
First, is q on the plane, P? We can check this by substituting the coordinates of q into the equation presented for P: 5·2–7·1+6·3=10–7+18=21, and 21 is not 10, so q is not on P.
My strategy for finding the distance was to write the parametric equations for a line through q perpendicular to P, see where the line intersected P, and then compute the distance from q to that point. So let me try to carry this out.

  • A line perpendicular to P through q We know that n=<5,–7,6> is a vector normal to P. So
    x=2+5t
    y=1–7t
    z=3+6t
    are parametric equations for the desired line.
  • The intersection of the line and the plane Take the parametric equations and plug them into the equation for the line:
    5x–7y+6z=10 becomes 5(2+5t)–7(1–7t)+6(3+6t)=10 which is (25+49+36)t=10–10+7–18 and this is 110t=–11 so t=–11/110=–1/10.
    What does this value of t mean? It is the value of t which, when substituted in the parametric equations, gives the intersection point. So:
    x=2+5(–1/10)=15/10
    y=1–7(–1/10)=17/10
    z=3+6(–1/10)=24/10
    The intersection point is (15/10,17/10,24/10).
  • The distance between q=(2,1,3) and the intersection point This is sqrt{(2–15/10)2+(1–17/10)2+(3–24/10)2}. This is sqrt(25+49+36)/10=sqrt(110)/10.

     
    Another way ...
    There are frequently different and equally valid solution strategies. Here is another way to do this problem: (2,1,3) is not on the plane 5x–7y+6z=10. I can find a point on the plane just by "guess" (as I did earlier, and I will guess for something convenient!): r=(2,0,0). Then the vector rq=<0,1,3> points from the plane to q. Now if I project rq onto the normal vector I will get a vector whose length is the distance from the plane to the point. This "projection" is just what I did last time when I found the parallel part of a vector. So n=<5,–7,6> is still a vector normal to the plane. So we just compute rq·n/||n||=(–7+18)/sqrt{110}=11/sqrt{110}, which is the same answer. This is more direct and more efficient and would be what I would do if I needed distances frequently.

    Parallel lines
    I guess two lines are parallel if they are distinct (that is, separate lines!) and if their directions are non-zero multiples of one another. So, for example, the lines

    Line A    Line B
    
    x=5+t     x=30+t
    y=3+2t    y=–9+2t
    z=–1+3t   z=–1+3t
    are parallel. Certainly their directions are multiples of one another (the multiple is 1). But why are they "distinct"? The point (5,3,–1) is on the first line. For it to be on the second line, uhhh ... 30+t=5, so t=–25. The second equation is then y=–9–(2)25=–59, and this is not 3. So (5,3,–1) is not on the other line.
    These are parallel lines.

    Parallel planes
    Two planes are parallel if they are distinct (that is, separate planes!) and their normal vectors are non-zero multiples of one another. For example, 2x+3y–z=56 and 4x+6y–2z=10 are parallel. Double the first equation and get the right-hand side of the second: that shows their normal vectors are non-zero multiples (the multiple is 2). But since twice 56=112 is not the same as 10, these planes are definitely distinct: different.

    Skew lines
    As dimensions increase, generally geometry gets more complicated. One phenomenon which does not occur in two dimensions is skew lines. A pair of lines is skew if they are not parallel and also do not intersect. So look at, say, the x-axis (where parametric equations could be x=t,y=0,z=0) and a line through (0,1,0) which is tilted with respect to the x-axis: x=t,y=1,z=t. These lines don't intersect since no point with second coordinate 0 is on the second line. They aren't parallel since <1,0,0>, the direction of the first line, is not a non-zero scalar multiple of <1,0,1>. So they are skew.
    Things get even worse as n, the dimension, increases. For example, it is possible to have pairs of skew planes (two dimensional objects) in R4.
     

    Exercise #2: the intersection of two (non-parallel!) planes
    The two planes 3x–y+z=4 and 4x+2y–z=7 are not parallel. I know this because the x/y/z coefficients in the equations are not (scalar) multiples of each other. They intersect in a line. How can we get parametric equations for this line?

    The planes are not parallel because the normal vectors (<3,–1,1> and <4,2,–1>, respectively) are not scalar multiples of one another. The planes are tilted differently. A Maple graph of these planes is shown to the right. I hope you can see the intersection is a line.

    A vector in the direction of the line will be perpendicular to both of these normal vectors. The cross product gives us such a vector: <3,–1,1>x<4,2,–1> is is <–1,7,10>.

    We also need a point on both planes. If x=0 then we must find y and z which satisfy both –y+z=4 and 2y–z=7. I added the equations and got y=11. The first equation immediately gives z=15. (When you first see this sort of thing it can be annoying, but there are many points on the line, and unless I am very unlucky [the line is perpendicular to the x-axis, for example!] I can just specify a value for any one of the variables and then solve for the others!) So (0,11,15) is on the line, and therefore a set of parametric equations for the line is
    x=0+–t
    y=11+7t
    z=15+10t.
    To the right is a picture of the two planes and the line of intersection (in blue, green, and red respectively).

    These pictures were drawn using the equations for the two planes and the line. I did play with them a bit to get the point of view shown, but not very much. I am not sure why the colors of the planes in the two pictures are slightly different.
    The principal commands used were plot3d, spacecurve, and display, which are all in the package plots. You will learn about this while doing the Maple labs.

     
    Another way ...
    One student suggested the following: take the "system" of equations
     3x-y+z=4 
    4x+2y-z=7
    and treat one variable, say z, as a parameter and then solve for the others. So this would happen:
     Original    z will be   Multiply first   Add the 
      system    a parameter  equation by 2   equations
     3x-y+z=4    3x-y=4-z      4x-2y=8-2z     8x=15-z
    4x+2y-z=7   4x+2y=7+z      4x+2y=7+z  
    
    so now I know that x=(1/8)(15–z) and since 3x–y=4–z we know y=–4+z–3x=–4+z–(3/8)(15–z). So we have x and y and z in terms of the parameter z (which could be rewritten at t).

    Most of you will later study linear algebra systematically, and what was just done will make more sense in the context of that subject. Here I wanted to use "tools" which were more native to 251 itself.

    Parametric equations for a plane
    Another way to describe a plane is parametrically. Pick a point on the plane, and pick two vectors with distinct directions pointing in the plane. Wait: let's look at an example. Suppose the plane contains the point p=(9,–5,1) and the vectors u=<3,–2,5> and v=<6,5,4> point along the plane (are parallel to the plane, really). Then for all numbers s and t, the points described by (9,–5,1)+su+tv describe a plane. I can break this up into components:
    x=9+3s+6t
    y=–5–2s+5t
    z=1+5s+4t
    We will deal with this alternative algebraic description of planes later when we discuss parametric surfaces. We can get back a vector normal to the plane by taking the cross product of u and v.
     

    Vector functions of a real number: space curves!!!
    Now the course moves on (chapter 13): we will deal with functions whose domain is all or part of R1 and whose range is R3. These are vector-valued functions of a scalar variable. The motivation is really the motion of points in space, and the analysis of the resulting paths (curves). There will be interactions between motion and geometry.

    Parametric curves: a nuisance?
    So I began with some irritating examples.

    1. The curve r(t)=<t,t,t>. This curve is a straight line. It goes through the origin, (0,0,0), and the path described is all of the line in the direction of the vector <1,1,1> (since <t,t,t>=t<1,1,1>). The motion described by this vector function (a triple of standard real-valued functions) is uniform rectilinear (straight-line) motion.
    2. The curve r(t)=<t3,t3,t3> is all of the straight line, but the kinetic aspect is very different. Motion is not uniform, and gets faster as |t| gets larger both positive and negative.
    3. Now the curve r(t)=<t2,t2,t2> is not too surprising. It is a half line or ray. Dynamically this represents the motion of a point starting way out in the first octant where all of x and y and z are positive, then coming in (slower and slower) to (0,0,0). Then the point turns around (?) and starts out into the octant again, retracing its path and going faster and faster.
    4. The curve r(t)=<sin(t),sin(t),sin(t)> is quite nasty psychologically if discussed immediately after the line. All of the coordinates are equal so every point on this curve is on the straight line. Some thought is needed to force yourself to agree that the "curve" is only a line segment from (–1,–1,–1) to (1,1,1) because sine's range is [–1,1]. The motion described by this vector function oscillates endlessly between the two points named.

    Distinguishing between geometric aspects of the path of a particle and the dynamic aspects of the particle will take up most of our next class. This won't be obvious.

    Some textbook problems
    I tried to discuss two problems from the textbook. These pictures are copied from page 743 of the text.


    Problem 5 of section 13.1
    Match the space curves in Figure 8 with their projections onto the xy-plane in Figure 9.
    Answer
    How can we "solve" this problem? I actually think most people can solve the problem, but I think describing the process is somewhat difficult. It is difficult because, for example, the pictures in Figure 8 are two-dimensional. And they are images of some ideal three-dimensional situation. What can we see? In Figure 8 (A) I see a straight line. I believe that the straight line will not be made "curvy" if we project into the xy-plane. Therefore the only candidate which matches is the non-curvy (!) picture (ii) in Figure 9. What else? Maybe the picture in (B) seems to be helical (more precise information about a helix later). That's sort of a motion around a cylinder whose axis of symmetry is, in this case, the x-axis. If we push this down and neglect the z coordinate, I think what we get is picture (i). This leaves (C) matching up with (iii). If you think about a particle moving around the path indicated in (C), maybe you see the xy-"squashing" of it moving back and forth along the parabolic path in (iii). Is this somehow "proof" that this was (iii)? We're told that the triples of pictures match up, and (C) and (iii) are what's left. It is certainly true that something like (C) could have a more complicated squashing than what is in (iii).

    Problem 6 of section 13.1
    Match the space curves in Figure 8 with the following vector-valued functions:
    (a) r1=<cos(2t),cos(t),sin(t)>
    (b) r2=<t,cos(2t),sin(2t)>
    (c) r3=<1,t,t>
    Answer
    Well, the linear formula is (c), and if we just consider the first two coordinates, the 1 and the t, that would seem to describe (ii), so I think that (c) is the algebraic version of (A). As for (b), the first two coordinates are t and cos(2t) which seem to describe (i). And if you consider the second and third coordinates of (b), they are cos(2t) and sin(2t). The sum of the squares of these is 1 (cos2+sin2=1) and the first coordinate in (b) just moves the point along, so we get the helix in (B). That leaves r1 in (a) to describe (C) and (iii). Let's consider the first two coordinates of (a), which are cos(2t) and cos(t). A trig identity you may/may not remember is cos(2t)=2(cos(t))2–1. If we take x as cos(2t) and y as cos(t), this becomes the equation x=2y2–1 which surely looks like (iii). And the third coordinate, sin(t), of (a) makes the image go up and down, up and down. So (a) is consistent with both (C) and (iii). I think this third curve and formula are a bit interesting and strange: the projection of the loop is a piece of a parabola!

    The twisted cubic
    The twisted cubic is the curve r(t)=<t,t2,t3>. It is one of the beginning curves shown to students in this subject, with the warning that here the pictures can be a bit deceptive. A Maple command created a 3-dimensional viewing box of the curve. I rotated this box in various ways and exported the images shown below.
    spacecurve(<t,t^2,t^3>,t=–5..5,color=black,axes=normal,thickness=2);

    The view from the z-axis
    Here we look down on the curve from high up on the z-axis. This has the effect of suppressing (?) or deleting the z-coordinate from the triple, and we just see the geometry of the first two coordinates: (t,t2). Of course, this is the parabola y=x2.
    The view from the y-axis
    Now delete the central variable. In the xz-plane, the curve is the collection of points (t,t3) and this is z=x3, which is, I hope, a fairly familiar cubic. And, indeed, if you orient the twisted cubic properly, then Maple shows you the displayed picture.
    The view from the x-axis
    This is picture which makes life difficult. Projected only the yz-plane, the collection of points (t2,t3) is the same as those points satisfying y3=z2. Now since z2 must be non-negative, any y's on this curve had better be in the right half of the yz-plane. And for each y>0, there are two z's (the positive and negative square roots). But the worst part is the behavior at the origin. z=+/–y3/2 has a horizontal tangent at (0,0): the curve has what's known as a cusp at the origin, a type of corner. It looks sharp, not smooth!
    An oblique view
    This "curve" actually does not have a corner in three-dimensions. The pictures already shown do not make this easy to see. To the right is sort of an oblique view of the curve as drawn by Maple. Although this picture seems to show a loop, the curve does not actually have any loops. The loopy look is a result of the angle I picked. The difficulties I discussed here are one reason the instructors would like you to be familiar with Maple -- use of it will help your intuition.

    A perspective on real life
    Frequently, similar difficulties have occurred in dissections of bodies, when slides of different cells are prepared. Depending on the angles of the slices, very different pictures are seen. This is a very famous and sometimes difficult problem in pathology: how to reconstruct 3 dimensional objects from slices. In this specific case, a "badly" chosen "slice" of the twisted cubic seems to show that the curve has a corner which it really does not -- motion along it is quite smooth (more about this next time).

    Robots and molecules
    People who want to understand the conformation (geometry) of biologically interesting macromolecules will need to learn how to look at curves in R3. Also, people who want to describe motion of robot arms necessarily need the ideas we will describe.

    QotD
    I asked people to sketch r(t)=<5cos(2t),5sin(2t),8t>. I remarked that this was a helix -- that is was a circular motion around a central axis combined with moving along that axis. I said that it could be inscribed in a circular cylinder, and asked what the radius of this cylinder was. I also asked for the distance between distinct loops of this helix. The image to the right was created mostly with two Maple commands:

    spacecurve(<5*cos(2*t),5*sin(2*t),8*t>,t=-2..4,numpoints=300,color
       =black,thickness=3,axes=normal);
    plot3d([5*cos(u), 5*sin(u), 8*v],u=0..2*Pi,v =-2..4,color=blue,axes 
       =normal,transparency =.95)
    I'm showing these commands not to impress you, because as you will see these are not very complicated instructions. Instead I'd like you to begin to be familiar with using Maple, and seeing what can be done. The helix is displayed (the black curve) on the surface of a cylinder shown in light blue.

    The central axis of this helix is the z-axis (because the sine and cosine are in the first two coordinates and the linear function is in the third or z-coordinate): the z changes steadily as t increases, while the x and y coordinates "spin" around the origin.
    The pair 5cos(2t) and 5sin(2t) tell me that the circular part of the helix has radius 5 (square and add these to get a constant, 25, which is the square of the radius requested). There is something tricky happening to get the (vertical, z) distance between two loops. Because the frequency of rotation is altered with 2t instead of t, we see that the output of the first two coordinates of the helix repeat when t is increased by Π not 2Π. Then the z increase gets multiplied by 8, so that the distance between two of the loops is 8Π. If you inspect the picture carefully, maybe you can convince yourself that the vertical separation between two of the loops is indeed about 8Π≈25. The vertical (z) units are displayed differently (scaled differently) from the horizontal (x and y) units.


    Tuesday, September 7, sections 12-14, and sections 15-17, lecture #3

    Things for students to think about
    The wonderful Rutgers schedule declares that Wednesday is Monday. Since all of our recitations are on Wednesday, this means you won't have a real recitation until two weeks into the course. Some stated reasons for recitations are enchanced opportunities to ask questions and see more problem solutions: this is really NOT GOOD. Therefore you should

    These QotD's, the very first!, asked students to apply dot product to diagnose perpendicular vectors (so the dot product should be equal to 0). Then some algebraic work needed to be done. For sections 12-14, this was solving two equations in two unknowns (with small integral coefficients). For sections 15-17, this was applying the quadratic formula to a degree 2 equation with small integer coefficients. Specific solutions are earlier in the diary and may be read.

    Math 251 students must be able to compute solutions to such questions without much effort. If you can't, then you are in trouble.

    The dot product and its basic properties
    Algebraic description If v=ai+bj+ck and w=di+ej+fk, then v·w=ad+be+cf. This is a number or scalar.

    Geometric description v·w=||v|| ||w|| cos(θ) where θ is the angle between the vectors v and w (and, of course, ||v|| and ||w|| are, respectively, the lengths of v and w).

    The dot product obeys the following algebraic rules which are not difficult to check but the details are tedious.

    1. If v and w are vectors then v·w is a scalar (in this course this means "a real number").
    2. (Commutativity) v·w=w·v. (You can see this is correct by looking at the pieces of v·w and w·v. For example, ad is the same as da.)
    3. (Linearity) Distributivity of vector addition over dot product (v1+v2)·w=(v1·w)+(v2·w). (Again, you can check that this is correct by doing algebra with the components. A similar statement is true about the second factor, since dot product is commutative.)
      Scalar multiplication distributes also (cv)·w=c(v·w (and for the second factor also).
    4. v·0=0.
    5. v·v=||v||2.

    But what do we know about cross product? I gave a geometric description. If v and w are vectors in R3, then vxw is another vector, not a scalar or number. We can specify it with direction and magnitude. The magnitude of vxw is ||v|| ||w|| sin(θ), the area of the parallelogram having edges v and w. The direction of vxw is perpendicular to the plane determined by v and w. The perpendicular direction is selected so that v, w, and vxw satisfy the right-hand rule.

     
    Problem 28 from section 12.4
    So we need to find vxw where v and w are vectors of length 3 in the xz-plane oriented as in the picture, and the angle θ between the vectors is π/6. When I first glanced at this problem I thought that there was not enough information supplied to get the answer. But let's think.
  • What is the direction of vxw? Please try to curl the fingers of your right hand from v to w. I think if you do, mentally at least (I tried this physically!) the thumb of your right hand will point in the direction of the negative y axis. The direction should be –j, a negative multiple of j.
  • What is the magnitude of vxw? This should be ||v|| ||w|| sine of the angle between the vectors. We are told that this angle is π/6 which has sine equal to 1/2. And each length is 3. So the magnitude of the cross product is (3·3)(1/2)=9/2.

    The direction should be backwards in y and the magnitude should be 9/2. The cross product therefore must be –(9/2)j.  

  • My goal is now to develop an algebraic formula for the cross product. Let me recall the the i,j,k multiplication table which we discussed last time. Here it is:

     x |  i  |  j  |  k
    --------------------
     i |  0  |  k  | –j  
    --------------------
     j | –k  |  0  |  i
    --------------------
     k |  j  | –i  |  0

    The algebraic formula for cross product
    I did the following awful computation, which you should never do:
    If v=ai+bj+ck and w=di+ej+fk, then vxw=
    (ai+bj+ck)x(di+ej+fk)=ad(ixi)+ae(ixj)+af(ixk)+bd(jxi)+be(jxj)+bf(jxk)+cd(kxi)+ce(kxj)+cf(kxk).
    I filled in all the values from the multiplication table above and got the following mess:
    vxw=ad(0)+ae(k)+af(–j)+bd(–k)+be(0)+bf(i)+cd(j)+ce(–i)+cf(0)=(bf–ce)i+(cd–af)j+(ae–bd)k

    This last formula can be expressed in a very neat way if you know about determinants. It turns out that

           | i j k | 
    vxw=det| a b c |
           | d e f |
    There is a very brief discussion of determinants in the textbook. I will probably evaluate this determinant most of the time by "expanding along the first row" but you may use any valid method. But before I do even a small harmless example, I should ask you:

    Why is what I've done correct?
    The definition I gave for cross product was geometric. Some of the properties of cross product are easy from the geometric definition. But I've just used linearity (or distributed the sum over the cross product), something like this: (v1+v2)xw=(v1xw)+(v2xw). Why is this true? This is not clear (!) to me at all from the geometric definition. Here is a paper which shows why you can distribute using geometry. Figure 8 on page 9 of this paper is a picture which should convince (is supposed to convince) you that cross product does have the desired algebraic property. I hope you will find it convincing. I find it difficult to really understand. But the result is correct, and the following equations involving cross product are correct.

    1. If v and w are vectors then vxw is a vector in R3 with three components.
    2. (Anticommutativity) v·w=w·v. (This is because the famous "right hand" points sort of backwards when the order is reversed.)
    3. (Linearity or distributivity of vector addition over scalar multiplication) (v1+v2)xw=(v1xw)+(v2xw). (A similar statement is true about the second factor.)
    4. v·v=0 always.
    5. The cross product is not usually associative, so don't believe that (vxw)xQ and vx(wxQ) are the same at all. (This is not just "theoretical" because we will do lots of elaborate computations involving cross products, and learning the rules they do and do not obey will help with discovering, verifying, and understanding short cuts.)
    Now onto a simple example
    We can try this on a pair of vectors. Let's see (this is a pair chosen not randomly at all!): what is w1xw2 if w1=<1,3,2> and w2=<3,–2,1>.
    What is the cross product of w1=<1,3,2> and w2=<3,–2,1>?
    So we compute the determinant
       | i  j k |
    det| 1  3 2 |
       | 3 -2 1 |
    This is i{3·1–2(–2)}j{1·1–2·3}+k{1·(–2)–3·3}. This is 7i+5j–11k.

    The QotD for sections 12, 13, and 14 was this:

    Find a and b so that v=<a,2,b> is orthogonal to w1=<1,3,2> and also is orthogonal to w2=<3,–2,1>.
    The dot product converts this into two linear equations in two unknowns. But the cross product just computed allows us to conclude that 7i+5j–11k is a vector perpendicular to both w1 and w2. There's only one direction perpendicular to both of these vectors, so 7i+5j-11k must be a scalar multiple of <a,2,b>. The multiple is (look only at the middle or j component!) 2/5. Therefore a=(2/5)7 and b=(2/5)(–11). This agrees with the solution using the dot product method.

    The picture to the right attempts to shown the geometric situation. w1 and w2 are in blue. The direction of the cross product, which is also the same as the direction of the desired vector v is shown in magenta.

    Knowing several ways of doing a problem or computation is often very useful. Sometimes one will work better than another.

    Algebraic descriptions of lines and planes

    Lines are discussed in the text on pp.689–690. Planes are discussed in section 12.5.

    Mostly I think I know what a line is and what a plane is. Here is a picture with one of both of them displayed. But what happens if we have several of them, and we want to deal very precisely with them? It is useful, then, to have algebraic ways of describing and computing with lines and planes.

    Specifying a line
    The following information will usually be the most convenient way to specify a line in this course:

    Then the line will be everything starting out at p going forward or backward in the direction of v. More precisely, if q=(x,y,z), then q will be on the line if the vector pq (from p to q) is a scalar multiple of v. This might be easier to describe with an example.

    Example
    If p=(3,9,5) and v=<4,–7,6>, then the vector pq will be <x–3,y–9,z–5>. Suppose t is a scalar. The one vector equation pq=tv becomes the following three scalar equations:
    x=3+4t
    y=9+–7t
    z=5+6t
    So these are parametric equations for the straight line determined by the data p and v. Notice where the 3 chunks of p's information go (no t's!) and where the 3 chunks of v's information (the direction) go (with the t's).

    Other parametric representations
    I then played with these equations for a while. For example, the equations
    x=3+12t
    y=9+–21t
    z=5+18t
    represent the same line. I just multiplied the direction v by 3. This doesn't change the actual geometric line which is parameterized by the equations: every point in "this" line is in the other line, and, similarly, the other way around. And
    x=3–4t
    y=9+7t
    z=5–6t
    represents also the same line: I multiplied the direction by –1 but still we will get the same total collection of points: the same straight line. And, perhaps more complicated:
    x=7+4t
    y=–2+–7t
    z=11+6t
    This is also the same line. Notice, please, that the direction vector is the same, but what I have changed is the "base point", (7,–2,11). But if you consider the original parameterization, and put t=1 in those equations, we will get (7,2,11). So, in fact, the descriptions sort of "overlay" one another: the v's are the same, and the we've just happened to start the parameterizations at different points on the same line. In the next class, we will consider in detail the kinetic aspect of the parameterization, but right now I tell you that the lines (the collection of all the points described by these triples of equations) are the same.

    There is a plethora of different parameterizations for the same straight line.
    Plethora officially means "A superabundance; an excess."

     
    Symmetric equations
    Some manipulation gives another way to represent the line.
    x=3+4t becomes t=(x–3)/4
    y=9+–7t becomes t=(y–9)/(–7)
    z=5+6t becomes t=(z–5)/6
    so that we have (x–3)/4=(y–9)/(–7)=(z–5)/6. Such a collection of equations is called the symmetric form of the line. As far as I know, we won't describe lines this way in this course.
     

    Two points determine a line
    The original work of Euclid is available, translated and illustrated, for free on the web. You may see here. This work has been done by Professor Joyce of Clark University. So, way back, thousands of years ago, Euclid decided that a line would be determined by two points. How can we use such information to get an algebraic description?

    Example
    Suppose p=(3,3,–9) and q=(–5,2,1). What are parametric equations for the line through the points p and q?
    The vector pq=<–5–3,2–3,–5–(–9)>=<–8,–1,4> is a vector in the direction of the line. We could use either p or q (I'll use p here) for a "base point" for the line. So we get the following parametric equations for the line:
    x=3+(–8)t
    y=3+(–1)t
    z=–9+(4)t
    The colors are an effort to show where to put the data in the parametric formulas.

    Specifying a plane
    In this course, the data specifying a plane will also turn out to be a point and a vector. What I'm about to describe may not look immediately useful, but it will be, really. So what we will need is

    I hope that a picture can be persuasive. There are many planes which pass through the point p. But if you add on the requirement that, at the point p, the vector n be perpendicular, exactly one geometric "candidate" persists, and that's the plane I'd like to describe algebraically.

    What condition(s) on x and y and z will guarantee that q=(x,y,z) is on the plane? We can look and conclude that the vector pq should be perpendicular to n. We can check perpendicularity easily with dot product. An example may help.

    Example
    Suppose p=(4,5,–7) and n is the vector <11,2,31>. The vector pq is <x–4,y–5,z–(–7)> and pq is perpendicular to n exactly when pq·n=0. And that is
    11(x–4)+2(y–5)+31(z–(–7))=0
    This is a fine equation. You don't need to "clean it up". Of course, it can be rewritten (if you must!) as
    11x+2y+31z=44+10–217=163.

    Again, there are many equivalent equations for one geometric plane since there are many vectors normal to one plane, and many points on one plane.

    QotD
    A line has parametric equations

    x=2t+1 
    y=-t+2
    z=3t
    and a plane is specified by 5x–y+2z=1. These objects intersect in a point called FROG. Find the coordinates of FROG.
    I suggested that the parametric equations for the line could be "inserted" into the plane equation. Then the t-value for the intersection could be determined, and, in turn, values for the coordinates of FROG could be found.


    Thursday, September 2, sections 12-14, and Friday, September 3, sections 15-17, lecture #2

    Who am I?
    I'm a faculty member in the Rutgers Math Department. Much of my professional work has involved differential equations involving functions of more than one variable. I actually like the content of this course, and I may get almost incoherent with excitement and pleasure when describing some of the subject matter. Certainly I will draw pictures that seem to mean something to me, but they may be totally incomprehensible to almost everyone else. I apologize in advance.

    Who are you?
    Many of the students in these sections are engineering students. Many engineering students (especially in Mechanical, Chemical, and Biochem) will go on to take Math 421 where all the material of this course is used. Other students are from such disciplines as chemistry, computer science, mathematics, meteorology, and physics. Certain biological science majors take this course, principally because they will take physical chemistry, and the language of several variable calculus is essential in pchem. Additionally, some students may be in this class for "fun" (if that's possible) but may be interested in a business career. It turns out that there's a huge developing field of mathematical finance and the ideas of this course are, again, essential tools.

    By the way, the real reason for my typing in all those names and other "stuff" on this list is to give you a better chance of setting up weekly study sessions with other students. Careful analysis has shown that such peer study groups increase success more than almost any other factor (including which instructors are assigned!) Please check your own entry in this list.

    What will we study?
    I tried to briefly discuss the background of the course. The link indicated has more information. We will cover essentially all of chapters 12 through 17 of the textbook. Detailed information is available here.

    So the stunt we pulled on Wednesday (giving the first formal "lecture" during the first recitation meetings) has given me additional time to explain very intricate stuff at the end of the course. I hope you will appreciate that later.

    The easy (?) product
    Suppose v=ai+bj+ck=<a,b,c> (the text uses these angle brackets, and I will try to use them but probably I will get excited and replace them with ordinary parentheses once in a while) and w=di+ej+fk. Then I'll define the dot product or the scalar product or the inner product (all of these phrases are used but I will mostly use "dot product") of v and w to be the number ad+be+cf.
    Our notation for this will be v·w. Some textbooks use (v,w) or <v,w>.
    This doesn't look too impressive, but it turns out to occur enormously frequently in many applications (so many, as I mentioned in class, that most computer languages have devoted special attention to how to evaluate it rapidly, and, indeed, the chips of many CPU's have special "microcode" to evaluate inner products efficiently). Why would such things occur in "real life"?

    The farmer's market
    New Jersey is the Garden State. In the summer and early fall, almost every Friday I go to a local farmer's market and buy fresh New Jersey fruits and vegetables. So the task facing the cashier when I come up, hands full of stuff, is to figure out what I owe. The logic of the cost computation might be tabulated as follows:

    Type of purchaseApplesEggplantGarlicOnionsPeppersPotatoesZucchini
    Price per pound$1.50$1$2.50$1.25$1.50$175¢
    Weight of item
    in pounds
    31.2.34251.2

    Now really look at the table. If you compute the total cost, you'd better not confuse the order of the items. Each row has an 7-tuple of numbers: this is an element of R7, the collection of all ordered 7-tuples of real numbers. Let's call the second row the cost vector, C, and the third row will be the weight vector, W. What will the total cost of this lovely purchase? Clearly it is C·W dollars:
    (1.5)(3)+(1)(1.2)+(2.5)(.3)+(1.25)(4)+(1.5)(2)+(1)(5)+(.75)(1.2).
    I'm not particularly interested in the number, more in the logic.

    A hobby or two?
    Everyone should have a hobby, something silly to occupy their time when they should be working. One of my hobbies, since I started teaching calculus, is to go through a day and try to identify every use of the derivative idea. There are lots and lots of them. Another hobby, perhaps even more fruitful (pun intended!) is to identify every darn use of inner product in a typical day. There are many uses. But I am supposed to concentrate on three dimensions, so let's get back to work.

    Looking at lengths again
    Start with v=ai+bj+ck so that ||v||=srqt{a2+b2+c2}. If w=di+ej+fk, then ||w||=srqt{d2+e2+f2}. If I draw the vectors v and w with their tails on the origin, then a vector, identified in the picture with "?", completes a triangle, going from the head of w to the head of v. Rhat vector isn't too mysterious. I know that w+?=v, so ?=v–w. In terms of components, we can write v–w=(a–d)i+(b–e)j+(c–f)k. The length of v–w is sqrt{(a–d)2+(b–e)2+c–f)2}. You can check easily that the length of v–w and the lengths of v and w are not related in any simple way.

    Really the squares of the lengths
    With all the square roots, maybe we should be looking at the squares of the lengths (square roots are difficult to deal with!). Also the algebra will be easier!
    Then ||v||2=a2+b2+c2 (this is the same as v·v, the dot product of v with itself)
    and ||w||2=d2+e2+f2
    and ||v–w||2=(a–d)2+(b–e)2+(c–f)2.
    We can "expand" the last term to get
    ||v–w||2=a2–2ad+d2+b2–2be+e2+c2–2cd+f2.
    That is exciting.

    The obstacle to making it work
    If you compare terms, the sum ||v||2+||w||2 is the same as ||v–w||2 except for –2ad–2be–2cf=–2(ad+be+cf). So computations with lengths and squares of lengths will have to include that "mess". Let me forget the –2, which isn't that essential. What an accident: the obstacle to making things ridiculously simple is (essentially) the dot product.

    The law of cosines lurks ...
    There's a formula which generalizes the formula in the Pythagorean Theorem. For the triangle shown above, here it is:
    ||v–w||2=||v||2+||w||2–2||v|| ||w||cos(θ)
    Here θ is the angle opposite the side v–w. This formula is actually not too difficult to verify. The green stuff is, of course, what you get from Pythagoras, and the red stuff can be thought of as a sort of "correction term", what should be done if θ is not a right angle and the two including sides (the sides on each side of θ) are not perpendicular.

    If θ is Pi/2, the cosine term is 0 and we get back Pythagoras. Here is the geometric view of the areas of the squares on the sides of a triangle.

    The geometry of the dot product
    Since we know that ||v–w||2=||v||2+||w||2–2||v|| ||w||cos(θ) and we know that ||v–w||2=a2–2ad+d2+b2–2be+e2+c2–2cd+e2=||v||2+||w||2–2v·w. We see that (cancelling the –2's)
    v·w=||v|| ||w||cos(θ) where θ is the angle between v and w.
    Notice that we can now see if v and w are not 0, then v and w are orthogonal (perpendicular, normal) exactly when v·w=0. (All three of those words are used, and I will use them almost interchangeably.)

    Determining an angle
    Suppose v=<3,2,–1> and w=<5,2,6>. What is the angle between v and w?
    We compute: v·w=15+4–6=13, ||v||=sqrt(14), and ||w||=sqrt(65). Therefore cos(θ)=13/[sqrt(14)sqrt(65)]. This gives approximately (calculator-assisted computation here!) 64.55 degrees or (better, I guess) 1.126 radians.

    Resolving a vector into perpendicular and parallel parts
    Let me continue using the v=<3,2,–1> and w=<5,2,6> from the previous question. I'd like to show you how to write v as a sum of two vectors, v|| and v where v is perpendicular to w and v|| is parallel to w. This will be useful several times in the course, and is also important in physics.

    How long is v||? If you look at the picture, the v|| is part of a right triangle. It is the adjacent leg and v is the hypoteneuse. θ is the angle between them. Therefore ||v||||=||v||cos(θ)=v·w/||w||. We did some of these computations just above, so ||v||||=13/sqrt(65). How can we get the correct direction? So we want a vector whose length is 13/sqrt(65) and whose direction is the direction of w. We can create a unit vector (a vector of length 1) in the direction of w: that will be w/||w|| which is <5,2,6>/sqrt(65). And now we adjust this unit vector by stretching it, to get [13/sqrt(65)]<5,2,6>/sqrt(65). This is about all I did in class, but let me finish things here. The vector is (13/65)<5,2,6>=<1,26/65,78/65>. Since v=v||+v, we know that v=v–v||=<3,2,–1>–<1,26/65,78/65>=<2,104/65,–143/65>.

    Checking the answer
    v should be perpendicular to w. So let's compute v·w=<2,104/65,–143/65>·<5,2,6>=10+(208/65)–(859/65)=(650+208–858)/65=(858–858)/65=0, so these vectors are indeed orthogonal.
    Comment I admit (informative to you and irritating to me) that I did all these computations by hand, and I had to do them three times to get them to come out correctly. The computation here occurs frequently in some elementary physics applications, and is the beginning of a rather profound and widely used algorithm in linear algebra called the Gram-Schmidt process.

     
    Color change?
    I'll write material with this background color if I just did not have time to discuss it in class but if I think it is useful enough that students might profit from seeing it.

    The dot product and its basic properties
    If v=ai+bj+ck and w=di+ej+fk, then v·w=ad+be+cf.
    The dot product obeys the following algebraic rules which are not difficult to check but the details are tedious.

    1. If v and w are vectors then v·w is a scalar (in this course this means "a real number").
    2. (Commutativity) v·w=w·v. (You can see this is correct by looking at the pieces of v·w and w·v. For example, ad is the same as da.)
    3. (Linearity or distributivity of vector addition over scalar multiplication) (v1+v2)·w=(v1·w)+(v2·w). (Again, you can check that this is correct by doing algebra with the components. A similar statement is true about the second factor, since dot product is commutative.)
    4. v·0=0.
    5. v·v=||v||2.

    Example with a little table
    Suppose we have four rambunctious ("Boisterous and disorderly") vectors: v1 and v2 and w1 and w2. Suppose also we have this little multiplication table for their dot products:

     · | w1 | w2
    --------------
     v1 | 3  | 7
    --------------
     v2 | 4  | –2
    Could we then compute something like
    (4v1–3v2)·(5w1–2w2)?
    Here I would expect people to distribute or take advantage of linearity on both the left- and right-hand sides. The process might go like this:
    (4v1)·(5w1–2w2)+(–3v2)·(5w1–2w2)
    (4v1)·(5w1)+(4v1)·(–2w2)+(–3v2)·(5w1)+(–3v2)·(–2w2)
    (4·5)v1·w1+(4·–2)v1·w2+(–3·5)v2·w1+(–3·–2)v2·w2
    (4·5)(3)+(4·–2)7+(–3·5)(4)+(–3·–2)(–2)
    And the answer seems to be .... 60–56–60–12=–68. I think this is correct. I make more errors in such computations when no one is watching me.
     

    Another product, introduced geometrically
    I defined the dot product by a rather simple algebraic formula. The cross product has a complicated algebraic formula, and maybe it is easier to begin by defining it geometrically.

    The cross product (also called the vector or outer product) takes two vectors, v and w, and produces vxw, another vector. Since this is a vector, it is specified by its magnitude and direction.
    Magnitude of vxw
    Look at the the vectors v and w. There is a parallelogram which has sides v and w. The area (a non-negative quantity) of that parallelogram is the magnitude of vxw.
    There is an easy formula for the area in terms of the lengths of the vectors and the angle between them (you get it from using the base times the altitude, where the base has length one of the vectors and the altitude is sine multiplied by the length of the other vector). The area is ||v|| ||w||sin(θ). You get this by multiplying the base by the altitude in the plane of the parellelogram.

    Direction of vxw
    Take your right hand and extend your thumb. Curl up your fingers. Insert your hand (I guess by thought, not physically) so that your fingers go from v to w. Then your thumb will be (sort of) pointed perpendicular to the plane containing v and w. That perpendicular direction is the direction of vxw.

    The only reason that such a weird product would be considered is that it is useful. We will see some geometric uses of it in this course and most of you will see uses in mechanics (torque), electromagnetism, fluid flow, etc.

    The i,j,k multiplication table
    So before anyone could yell too much I computed the cross product "multiplication table" for i and j and k. This wasn't too hard, although getting the directions correct involved some thought and some physical contortions. These are unit vectors and all mutually perpendicular. If the vectors in the product are the same, then the parallelogram involved collapses to a line segment and has no area. If the vectors are different, then the parallelgram is a square with 1 unit sides, and has area 1. The important thing is the sign of the answer and its direction. I tried to get that correct. If you were not in class or you were confused, I urge you to try to get some of the entries in this table yourself.Then the cross product in terms of components is

     x |  i  |  j  |  k
    --------------------
     i |  0  |  k  | –j  
    --------------------
     j | –k  |  0  |  i
    --------------------
     k |  j  | –i  |  0

    Weird stuff
    Things to notice:
    ixj=k and jxi=–k. The cross product is not necessarily commutative. In fact, the cross product is anti-commutative:
    vxw=wxv always.

    (ixj)xj=kxj=–j and ix(jxj)=ix0=0. The cross product is not necessarily associative.
    This means that you can't necessarily rearrange or regroup cross products. This is annoying. The cross product probably does not resemble many other products you have seen.

    QotD
    During almost all lectures I will ask a Question of the Day. These will be informal quizzes designed to help students and help me learn about students. The questions will be (I would like to think!) easy and fast. I will read and comment on the answers. Students get full credit for any answer, so the answer "Giraffe" would always get full credit. Students may work with other students. What are my secret and horrifying aims in doing this?

    QotD for sections 12, 13, and 14
    Find a and b so that v=<a,2,b> is orthogonal to w1=<1,3,2> and also is orthogonal to w2=<3,–2,1>.
    Here we translate the requirements as v·w1=0 and v·w2=0. The first equation becomes a+6+2b=0 and the second becomes 3a–4+b=0.
    Yes, Math 251 students should be able to
    solve two linear equations in two unknowns.

    Well, you can guess the answers (regrettably, I do that sometimes!) but there turns out to be a systematic process (Gaussian elimination) to discover if there are any at all, and, if there are, to list them efficiently. Here if
    a+6+2b=0 and 3a–4+b=0 then (multiplying the first equation by 3) the system becomes
    3a+18+6b=0 and 3a–4+b=0 then (subtracting the second equation from the first) we get
    22+5b=0 so that b=–22/5. Then substituting this value of b into the first equation, we get
    a+6+2(–22/5)=0 so that a=–6+44/5, a fine answer, or, "simplifying", a=14/5.

    QotD for sections 15, 16, and 17
    Find any and all values of a so that v=<a,1,a2> and w=<3,2,–1> are orthogonal.
    So we need v·w=0 which is 3a+2–1–a2=0.
    Yes, Math 251 students should be able to
    use the quadratic formula to find roots of
    a quadratic equation.
    .

    Well, if you run into lots of natural quadratic equations that can be factored easily to get the roots, then please let me know. This will happen only rarely! Solutions of 3a+2–1–a2=0 are [–3+/–sqrt{32–4(–1)(2)}](–2). This is a fine answer and can be left as is. If you wish to "simplify" you would get [3+/–sqrt(17)]/2.


    Wednesday, September 1, lecture #1

    The "first lecture", according to the syllabus, was given by the recitation instructors, Mr. Bouch and Mr. Nanda. I thought this could be the best use of the time since now we have an additional lecture to discuss some of the much more complicated material occurring later in the course.

    Diary
    I will try to write a diary for this course, since some students have told me it is helpful. I will use lots of material from the diary I wrote the last time I taught 251. This is not "plagiarism". I wrote the material, I acknowledge using it, and I've given myself permission. For your information, it isn't usually the text that takes much time, but rather the darn pictures. Lots of time is needed to create good pictures!

    What follows is my diary entry from the last time I taught the course. I hope it represents the ideas which were covered in the first meeting by Mr. Bouch and Mr. Nanda.

    We began with a "review" of analytic geometry of one and two and even three dimensions. I put the word in quotes because most students who have taken courses in applied science and engineering have already seen the material we will discuss in 2 or 3 or even 4 other courses! The poor math majors may never have seen this before so they especially should read the text.

    R1
    This is supposed to be the real line. There's an origin and a unit length, and the conventional choice is to make the positive direction go off to the right. Each point on the line corresponds to a real number. The distance between points on the line which correspond to the real numbers a and b is defined to be |a–b|.

    R2
    And here is the plane (I don't think I've ever heard anyone call it the "real plane"). The simplest situation is what's shown here. Two straight lines as axes, perpendicular to each. Every point corresponds to a unique ordered pair of real numbers. There is a set unit length on each coordinate axis, and almost always right is positive and up is positive. Later in the course we will consider situations where the coordinate axes are not necessarily perpendicular. And even where what corresponds to the axes are not straight. I hope that tilted axes could make sense if, for example, you were interested in looking at crystals which did not have rectangular symmetry. I used Pythagoras to deduce the distance formula giving the distance between two points in the plane.

    R3
    In three dimensions, the standard geometric situation is specified by three mutually perpendicular lines. Points will correspond to ordered triples of real numbers. In almost every situation you are likely to encounter, these axes will be right-handed. Right-handed means ... means ... means ... it means what I sketched to the right (heh heh). The three dimensional world is complicated, and you will see that the choice of right-handedness has some interesting consequences.

    Chirality or handedness
    A discussion of Chirality is linked. Here is a short discussion of parity in chemistry and physics. And here is a Wikipedia article on enantiomers, which are compounds that are non-superimposable mirror images of one another. This article is a bit more technical, but has many examples of pharmaceutical products which are mirror images.

    Perilous pictures
    Here is my attempt to draw the point with coordinates (3,2,–1). The green "path" with 's sort of tells how to find the point. I walk (?) from (0,0,0) to (1,0,0), then to (1,3,0), and finally to (1,3,–2). Each is supposed to represent one unit of the path. I think that the complications of perspective and reducing the "object" from three dimensions to a two-dimensional representation may make even such a simple picture difficult to understand. If you weren't told that the point was supposed to be (1,3,–2), would you have guessed those were the coordinates? Sigh.
    I will try to draw good pictures, but I won't always succeed. And even if I draw pictures which I believe are suitable, you may not agree. Pictures are complicated and sometime personal.
    Some simple geometry and equations
    What points in R3 have y=2? Certainly (0,2,0), on the y-axis, is the only point on the y-axis with y=2. We can move up and down (changing the z-coordinate) and sideways (changing the x-coordinate). We can even change both. The collection of points we get is a plane, perpendicular to the y-axis through the point (0,2,0). An attempt at a picture is to the right. You don't need to agree that this is a good picture.
    In this course I will probably regard the following technical words as synonymous (the words mean the same) most of the time:
    perpendicular    orthogonal    normal
    What points in R3 have z<5? This is a an open half-space. The plane z=5 is the boundary of this region and the region does not include the boundary. Later in the course I'll try to discuss the use of "open" and "boundary" in more detail. Here the boundary is a plane normal to the z-axis through the point (0,0,5). The plane, however, is not included in the region specified by z<5. What I've attempted to show in the picture is an open half-space.
    Let's imagine a brick in R3 with its sides parallel to the xy and yz and xz planes. Maybe this could be called a right-angled parallelopiped. Suppose one corner (also called a vertex) has coordinates (a,b,c) and the diagonally opposite corner has coordinates (d,e,f).
    Now let's look at the corner displayed in the accompanying picture, the corner illustrated with . What are the coordinates of that point?
    If you can "see" what the picture is supposed to show, then the designated corner is just moved "out" parallel to the direction of the y-axis. The x-coordinate doesn't change at all, and the height from the xy-plane (which is the z-coordinate) also doesn't change. Therefore the coordinates of are certainly (a,?,c). What about the second coordinate of the point? If you look at the picture with me, you can see that the entire face or side of the solid between and the point (d,e,f) has constant y-value: it is perpendicular to the y-axis. Therefore the mystery middle coordinate must be the same as the middle coordinate of (d,e,f), and so =(a,e,c).
    Now let us play the same game with another vertex (or corner). So the designation of has been moved to the corner shown. What are the coordinates of this point? This point is on the same face or side of the solid so it certainly shares the second or y-coordinate with both (d,e,f) and (a,e,c), so =(?1,e,?2). The third coordinate, ?2, must be c since the point we're looking at is certainly on the bottom face of the solid. The first coordinate might be the most puzzling, but both (d,e,f) and are on the "back" face (that is part of the plane x=d) of the solid, so the x-coordinate we're looking for is d. Therefore this is (d,e,c).
    A good exercise for you is to figure out the coordinates of some of the other corners of the brick.
    Now I'd like to compute the length of some of the edges of the brick. For example, what is the length of the line segment connecting (a,b,c) and (a,e,c)? This is the distance between these two points. Notice that the only coordinate which is varying is the middle one. This side is a line segment which is parallel to the y-axis. We can measure length along it only by thinking about the one-dimensional distance between b and e. That distance is |b–e|. The reason for the absolute value signs is that distance is supposed to be non-negative.
     
    We can try to do the same thing with the edge connecting the points with coordinates (a,e,c) and (d,e,c). Here the only coordinate difference is in the first, x-, coordinate. So distances again should be measured as if they were in one dimension, along the parallel x-axes. The distance we want is |a–d|.
     
    Now we can be even more bold. (Bold for a math class.) We can find the distance from (a,b,c) to (d,e,c). Look carefully and see that there is a right triangle whose hypotenuse is the distance wanted, and whose "legs" have distances we already know. Then Pythagoras declares that the length of the hypotenuse is
    sqrt(|b–e|2+|a–d|2)
    If you are somewhat alert this result is not too surprising. The bottom face of this brick is where z=c. The formula written is just the ordinary two-dimensional distance between the points (a,b) and (d,e).

     
    I'm not going to try to put that formula in the picture. I will just label the distance Blah. Now I want the distance along one of the main diagonals of the brick, from (a,b,c) to (d,e,f). There is, as shown, another right triangle. The hypotenuse is the distance I want, and the two legs have distances we know: one is Blah and the other is ... the other is: the same ideas as before (since the endpoints differ in only one coordinate) tell me that the distance is |c–f|. Therefore (again Pythagorus) the distance between the farthest apart corners is
    sqrt(Blah2+|a–d|2)
    But let me write it out in all of its glory (or gory [details, that is!]):

    sqrt({sqrt(|b–e|2+|a–d|2)}2+|a–d|2)
    Here I inserted the formula we previously got for Blah. But lots of what is written in this formula is superfluous. The square of a square root is always the number. We don't need absolute values if we are square a number. So therefore we can make a spontaneous (!) definition:

    The Euclidean distance
    The (Euclidean) distance between (a,b,c) and (d,e,f) is sqrt((a–d)2+(b–e)2+(c–f)2).

    A bunch of comments

  • First, math definitions shouldn't (and generally aren't) written at random. They ideally should be supported by examples and "intuition". Here the word intuition really means looking at lots of situations and extracting the important common ideas.
    In this course you'll observe an enormous storm of definitions. Almost all of them are not motivated by anything resembling "theoretical" considerations. The definitions were created to help discuss various real physical and geometrical situations. So the fact that the math is widely used in so many applications is not an accident -- it was arranged that way!
     
  • Second, the definition actually shows you how to define distance between points in R17, if you would ever want to. The definition should be something like the square root of the sum of the squares of the differences between the respective coordinates of the points. And this actually (for many purposes) is a good definition. The distance between two points is 0 exactly when the points are the same, and the distance between p and q is the same as the distance between q and p. Math majors who have taken Math 300 might recognize the skeleton of a definition using mathematical induction. But I can't talk about this here.
     
  • I remark that the distance defined is called the Euclidean distance because there are other ways of defining "good" distances, and some of them are useful in other contexts. Students who study digital signal processing will find out that the Euclidean distance is perhaps not always the most useful distance when comparing signals. In this course, the Euclidean distance will be the only one used, so I will almost always omit the adjective "Euclidean". What is nice about the Euclidean distance, though, is that it is very computationally neat, and leads to the method of least squares, which is a very commonly used method of fitting "curves" (function descriptions) to data points.

    A sphere of radius 3
    What algebraic condition on a point (x,y,z) is equivalent to the geometric statement that the point lies on a sphere of radius 3 centered at (0,0,0)? This means that the distance from (x,y,z) to (0,0,0) should be 3, or sqrt((x–0)2+(y–0)2+(z–0)2)=3 which of course is the same as x2+y2+z2=9.

    Completing the square to "recognize" a sphere
    We could of course "reverse" the process if we're given an equation (or at least an equation of the correct form). So the equation
    x2–3x+y2+4y+z2–7=0
    represents a sphere. What is its center and its radius? The key algebraic maneuver here is completing the square, and everything works very much as in two dimensions.
    x2–3x → x2+2(–3/2)x → x2+2(–3/2)x+(–3/2)2–(–3/2)2 → (x–3/2)2–(–3/2)2
    y2+4y → y2+2(2y) → y2+2(2y)+22–22 → (y+2)2–22
    We don't have any term with z to the first power. So the equation becomes:
    (x–3/2)2–(–3/2)2+(y+2)2–22+z2–7=0
    which is the same as
    (x–3/2)2+(y+2)2+(z–0)2=(–3/2)2+22+7
    and we can "read off" that the center is (3/2,–2,0) and the radius is sqrt((–3/2)2+22+7). It is easy to make mistakes with minus signs when identifying the center. It is also easy to forget to take the square root when identifying the radius. Oh well.
    If we had different (but positive) numbers in front of the squares (x2 and y2 and z2) then we'd get an egg-shaped object, an ellipsoid. We'll see more about this later.

    Vectors
    A vector is a directed line segment. It is used as a mathematical model for any quantity which has both magnitude and direction. Here are some quantities which are vectors and some which are not:

    Some vector quantities
  • Acceleration, velocity, displacement For velocity, for example, the length of the vector corresponds to speed: "I am traveling at 25 miles per hour" (in metric, I think that is 78 hectares per wombat). Of course the direction of the vector corresponds to the direction of motion: "I am traveling at 25 miles per hour North by Northwest" (in the metric system, this direction is called "Colder").
  • Force Another basic example of a vector. The magnitude is the amount of oomph in the force (I am giving up on units) while the direction is the direction the force is pushing in.
  • Moment, torque, etc. You'll meet lots of vector quantities.
  • Things which aren't vectors
  • Mass (=energy, according to our good friend Albert).
  • Temperature

    Quantities which are not vectors have the strange adjective, scalars. So both Mass and Temperature are scalar quantities, and are measured by numbers (both positive and negative) and have no intrinsic direction.

  • Equal vectors
    A vector can also be specified by its head and tail. Two vectors will be called equal (maybe the official word is equivalent) if when the tail of one of them is moved to the tail of the other, then their heads are in the same place.

    Vectors are used to do algebra in more than one dimension. This is because algebra has really been successful, and the interaction between algebra and geometry has paid off in both directions. Therefore we'll need to add and multiply vectors. Efforts to multiply run into serious difficulties, as we will see next time. Addition is neat and "everything" works. As I mentioned in class, the generally accepted definition for vector addition models some situations which can be experienced and measured in "real life" with forces, velocities, etc. The word resultant is sometimes used in connection with this definition.

    Definition of vector addition
    First suppose there are two vectors, v and w, which are roaming, free and happy in R3. We take the vector w and drag it so that the tail of w is at the same point as the head of v. The vector v+w is now defined using the geometric display we just arranged. v+w the vector whose tail is at the tail of v and whose head is at the head of w.

    Properties of vector addition

    1. Reality Addition (superposition, resultant) of vector quantities in physical "reality" is modeled by vector addition. As I mentioned in class, there probably are many ways we could define vector addition. This way is used because it accurately models certain physical situations. Please keep this in mind!
    2. Commutativity v+w=w+v for all pairs of vectors v and w.
      This can be proved from the geometric definition by drawing a parallelogram which has one pair of sides translates of v and the other pair of sides translates of w. Anyway, the order in which you add vectors doesn't matter.
    3. Associativity (v+w)+u=v+(w+u) for all triples of vectors v, w, and u.
      I think this can be proved by drawing some parallelopided (three-dimensional analogue of a parallelogram) with sides gotten form v and w and u: but the diagram would almost be more annoying than just thinking about it. The important consequence is that you can group adding vectors in any way you want, and the result will be the same.

    Zero and minus
    The zero vector has its head equal to its tail. If v is a vector, then –v is the vector whose length is v's but whose direction is the reverse: so the head of –v is the tail of v and the tail of –v is the head of v. Huh. Say that fast.
    I am especially proud of the image of the zero vector in the picture to the right. I wanted to show the special beauty of the zero vector. I tried several different angles. (This is a joke!)

    Then v+(–v)=0, and v+0=0 etc. for all vectors.

    Scalar multiplication
    In this course the scalars will be real numbers. Scalars are things that multiply vectors. Many of the students in the course will see later situations where the scalars are complex numbers. In the case of the vectors arising in computer science and electrical engineering, collections of scalars occur which are much less familiar.

    If c is a scalar and v is a vector, then the vector cv is defined by the following:
    If c>0 the direction of the vector cv is the same as the direction of v, and the length of cv is the length of v multiplied by c.
    If c=0 then cv is the zero vector.
    If c<0 the direction of the vector cv is the same as the direction of v and the length of cv is the length of v multiplied by |c|. (The absolute value makes sure that lengths stay positive.)
    Here are pictures of v and –v and 3v and –2v.
    Important vectors
    Some vectors are more important than others, especially if you have a coordinate system. The vectors i and j and k are vectors of length 1 (such vectors are called unit vectors) which are parallel to the coordinate axes x and y and z, respectively.
    Now take any vector v and move it so that the tail of v is at the origin, (0,0,0). Then the head of v will be at some point, (a,b,c). If you think about the geometry I hope you can convince yourself that v=ai+bj+ck. We have written v as a sum of its components.
    The textbook also uses the notation v=<a,b,c> for this.
    Vector addition in terms of components
    Since vector addition is commutative and associative, and certainly (c1+c2)(any vector)=(c1)(that vector)+(c2)(that vector), we see clearly that if v=ai+bj+ck and w=di+ej+fk, then v+w=(a+d)i+(b+e)j+(c+f)k.
    The components of the sum are the sum of the respective components. If you know the components, it is easy to compute vector sums and scalar multiples of vectors.

    The head and the tail produce the vector (algebraically)
    Suppose the point P has coordinates (x1,y1,z1) and the point Q has coordinates (x2,y2,z2). Then the vector from P to Q (so P is the tail and Q is the head) is just (x2–x1)i+(y2–y1)j+(z2–z1)k. I hope that you can draw a picture convincing yourself of that, or you can look in the text.
    Example The vector from P=(1,2,–3) to Q=(–5,6,2) is (–5–1)i+(6–2)j+(2–[–3])k=–6i+4j+5k. It is easy to make mistakes with signs in this computation.

    How long is a vector?
    I will use length and norm and magnitude to mean the same thing about a vector: its length. If v=ai+bj+ck, then we can think of the tail of v as sitting at (0,0,0) and the head of v, at (a,b,c). The length formula we got then says that the length must be sqrt(a2+b2+c2). So if you know the components, the magnitude can be computed. In the textbook we are using the magnitude is written ||v||. Please note that in some books and some situations, this quantity is just written |v|.

    NO! NO, NO, NO!!!
    Suppose that v=<1,2,3> and w=<–2,3,2>. Then v+w=<–1,1,5>. And ||v||=sqrt(14) (approximately 3.74), ||w||=sqrt(17) (approximately 4.12), and ||v+w||=sqrt(27) (approximately 5.19). In general, these numbers need not be related in any simple fashion, aside from the Triangle Inequality (see the textbook, please).

    Next time I will explore how these lengths can be explained, as we discuss multiplication of vectors.

    Start reading chapter 12, please. Do the problems.


    Maintained by greenfie@math.rutgers.edu and last modified 8/2/2010.